इंदौर शाखा: IAS और MPPSC फाउंडेशन बैच-शुरुआत क्रमशः 6 मई और 13 मई   अभी कॉल करें
ध्यान दें:

प्रारंभिक परीक्षा


प्रारंभिक परीक्षा

प्रारंभिक परीक्षा, 2022: एक विश्लेषणात्मक अध्ययन

  • 06 Jun 2022
  • 209 min read

1. निम्नलिखित युग्मों पर विचार कीजिये:

अशोक के प्रमुख शिलालेखों के स्थान वह स्थान जिस राज्य में हैं
1. धौली ओडिशा
2. एर्रगुडी आंध्र प्रदेश
3. जौगड़ मध्य प्रदेश
4. कालसी कर्नाटक

उपर्युक्त युग्मों में से कितने सही सुमेलित हैं?

(a) केवल एक युग्म     

(b)  केवल दो युग्म

(c) केवल तीन युग्म    

(d)  सभी चारों युग्म

उत्तर: (b)

व्याख्या:

  • धौली ओड़ीसा का एक महत्त्वपूर्ण प्रारंभिक ऐतिहासिक शहरी केंद्र है। धौली में पाए गए पुरातात्विक अवशेषों ने इसकी प्राचीनता का पता तीसरी शताब्दी ईसा पूर्व, विशेष रूप से अशोक के समय से लगाया। धौली ऐतिहासिक महत्त्व का है क्योंकि सम्राट अशोक के प्रसिद्ध शिलालेखों में से एक यहाँँ स्थित है। अत: युग्म 1 सही सुमेलित है।

https://magazines.odisha.gov.in/Orissareview/july2006/engpdf/49-53.pdf 

  • 2013 में भारतीय पुरातत्त्व सर्वेक्षण (ASI) ने आंध्र प्रदेश के कुर्नूल ज़िले में गूटी-पाथिकोंडा रोड पर एरागुडी के पास अशोक रॉक एडिट साइट की सुरक्षा पर जोर दिया। अत: युग्म 2 सही सुमेलित है।
    • शिलालेख मौर्य सम्राट अशोक (269-231 ईसा पूर्व) की महत्त्वपूर्ण संपदाओं में से एक थे, जिनमें वृहद् और छोटे शिलालेख शामिल हैं।

https://www.thehindu.com/news/national/andhra-pradesh/asi-to-develop-ashoka-rock-site-as-tourist-spot/article4766897.ece 

    • जौगड़ा एक प्राचीन किला है जो कलिंग प्रांत की मौर्य गढ़वाली राजधानी के रूप में कार्य करता था। जौगड़ा ओडिशा के गंजम ज़िले में बेरहामपुर और पुरुषोत्तमपुर शहरों के निकट स्थित है।
      • जौगड़ा ओडिशा एक अन्य स्थान है जहाँँ अशोक का वृहद् शिलालेख मौजूद है, इसे कलिंग शिलालेख भी कहा जाता है। अत: युग्म 3 सुमेलित नहीं है।

    https://odishatourism.gov.in/content/tourism/en/blog-details.html?url=jaugada-Ashokan-major-rock-edict-in-odisha 

    • जब अशोक ने बौद्ध धर्म को अपनाया उस दौरान कालसी स्थित शिलालेख में युद्ध के बाद अशोक के मानवीय दृष्टिकोण को लिपिबद्ध किया गया। कालसी उत्तराखंड और हिमाचल प्रदेश के बीच बफर ज़ोन में स्थित है। अत: युग्म 4 सुमेलित नहीं है।

    https://timesofindia.indiatimes.com/travel/uttarakhand/kalsi/ps36300328.cms 

    अतः विकल्प (b) सही है।

    https://www.drishtiias.com/to-the-points/paper1/mauryan-art-and-architecture-part-1

    2. निम्नलिखित युग्मों पर विचार कीजिये:

    राजा   राजवंश
    1. नान्नुक चंदेल
    2. जयशक्ति परमार
    3. नागभट द्वितीय गुर्जर-प्रतिहार
    4. भोज राष्ट्रकूट

    उपर्युक्त युग्मों में से कितने सही सुमेलित हैं?

    (a) केवल एक युग्म

    (b)  केवल दो युग्म

    (c) केवल तीन युग्म 

    (d)  सभी चारों युग्म

    उत्तर: (b)

    व्याख्या:

    • चांदला या चंदेल मध्य भारत का एक भारतीय राजपूत वंश था। उन्हें भारतीय इतिहास में लोकप्रिय रूप से चंदेल या जेजाकभुक्ति वंश कहा जाता था।
    • महोबा खंड के किंवदंतियों के अनुसार, चंदेल परिवार की उत्पत्ति चंद्रमा (हिंदी में चंद्र कहा जाता है) और हेमवती के मिलन से हुई थी।
    • 954 ई. के खजुराहो शिलालेखों के अनुसार, चंदेल वंश के प्रथम राजा नन्नुक ऋषि चंद्रत्रेय के वंशज थे जो प्रसिद्ध वैदिक ऋषि अत्रि के पुत्र थे। अत: युग्म 1 सही सुमेलित है।

    https://mahoba.nic.in/history/ 

    1

    • जयशक्ति का संबंध चंदेल वंश से था न  कि परमार वंश से। अत: युग्म 2 सुमेलित नहीं है।

    https://www.khajuraho-india.org/chandela-dynasty.html

    • गुर्जर-प्रतिहार राजवंश, मध्ययुगीन भारत के दो हिंदू राजवंशों में से एक था। हरिचंद्र के वंशजों ने 6वीं से 9वीं शताब्दी ईस्वी के दौरान, आमतौर पर सामंत के दर्जे के साथ, मंडोर, मारवाड़ (जोधपुर, राजस्थान) में शासन किया। 8वीं से 11वीं शताब्दी के दौरान नागभट्ट वंश ने पहले उज्जैन और बाद में कन्नौज में शासन किया। अन्य गुर्जर वंश मौजूद थे, लेकिन उन्होंने प्रतिहार उपनाम ग्रहण नहीं किया।
      • 9वीं शताब्दी की शुरुआत में हुए जटिल युद्धों - जिसमें प्रतिहार, राष्ट्रकूट और पाल शामिल थे - में नागभट्ट द्वितीय ने एक महत्त्वपूर्ण भूमिका निभाई।
      • नागभट्ट द्वितीय उत्तर भारत का सबसे शक्तिशाली शासक बना और कन्नौज में अपनी नई राजधानी स्थापित की। लगभग 833 ई. में नागभट्ट द्वितीय का उत्तराधिकारी उसका पुत्र रामभद्र था। अत: युग्म 3 सही सुमेलित है।

    https://www.britannica.com/topic/Gurjara-Pratihara-dynasty 

    • मिहिर भोज परमार वंश का राजा था। अत: युग्म 4 सुमेलित नहीं है।

    https://dhar.nic.in/en/bhojshala/

    3. प्राचीन दक्षिण भारत में संगम साहित्य के बारे में, निम्नलिखित कथनों में कौन-सा एक, सही है?

    (a) संगम कविताओं में भौतिक संस्कृति का कोई सन्दर्भ नहीं है।

    (b) वर्ण का सामाजिक वर्गीकरण संगम कवियों को ज्ञात था।

    (c) संगम कविताओं में समर शौर्य का कोई सन्दर्भ नहीं है।

    (d) संगम साहित्य में जादुई ताकतों को असंगत बताया गया है।

    उत्तर: (b)

    व्याख्या:

    • संगम साहित्य के रूप में जानी जाने वाली कविताओं का संग्रह छह शताब्दियों में लगभग 300 ईसा पूर्व से 300 ईस्वी तक तमिलों द्वारा विविध सामाजिक पृष्ठभूमि में निर्मित किया गया था।
      • ये कृतियाँ आरंभिक तमिल संस्कृति के साथ-साथ दक्षिण भारत और भूमध्यसागरीय, पश्चिम एशिया तथा दक्षिण पूर्व एशिया के बीच व्यापार संबंधों पर प्रकाश डालती हैं।
    • आरंभिक भारतीय साहित्य में संगमकालीन लेखन संभवतः अद्वितीय है, जो लगभग पूरी तरह से धार्मिक है। इसकी कविताएँ दो मुख्य विषयों से संबंधित हैं: पहले पाँच संग्रह प्रेम (अकम) पर आधारित हैं और अगले दो संग्रह वीरता (पुरम) पर आधारित हैं, जिसमें राजाओं की प्रशंसा और उनके द्वारा किये गए कार्य शामिल हैं।
      • कई कविताएँ विशेष रूप से वीरता (योद्धा नैतिकता से संबंधित), ऊर्जावान और उत्साह प्रदर्शित करती हैं तथा भारत के अन्य प्रारंभिक एवं मध्यकालीन साहित्यों की तुलना में साहित्यिक अतिश्योक्ति से पूरी तरह मुक्त हैं।
      • नायकों और संरक्षकों की प्रशंसा वाला बर्दिक साहित्य होने के कारण समाज और अर्थव्यवस्था के विभिन्न पहलुओं पर इसकी चिंता स्वाभाविक थी।

    https://en.unesco.org/silkroad/sites/default/files/knowledge-bank-article/sangam_literature_as_a_source_of_evidence_on_indias_trade_with_the.pdf 

    • संगम साहित्य पवित्र या जादुई शक्तियों में एक विश्वास को दर्शाता है जिसे अनकू कहा जाता है जो विभिन्न वस्तुओं में रहने वाले थे।

    http://surl.li/cdrpp

    • "संगम" साहित्य किसी विशेष सामाजिक या धार्मिक समूह का उत्पाद नहीं है और न ही इसे किसी शासक अभिजात वर्ग द्वारा दरबारी साहित्य के रूप में प्रायोजित किया गया था।
      • लगभग 600 वर्षों की लंबी अवधि में विभिन्न समय बिंदुओं पर रचित और विभिन्न स्तरों के व्यक्तियों - राजकुमारों, सरदारों, किसानों, व्यापारियों, कुम्हारों, लुहारों, बढ़ई और ब्राह्मणों, जैन एवं बौद्धों द्वारा लिखित, कविताएँ विषम सामाजिक समूहों से संबंधित हैं। (अतः वर्ण का सामाजिक वर्गीकरण ज्ञात था)।

    https://www.britannica.com/art/shangam-literature 

    • अतः विकल्प (b) सही है। 

    https://www.drishtiias.com/to-the-points/paper1/sangam-age-1 

    4. किसके राज्यकाल में ‘‘योगवाशिष्ठ’’ का निज़ामुद्दीन पानीपति द्वारा प़ारसी में अनुवाद किया गया?

    (a) अकबर            

    (b)  हुमायुँ

    (c) शाहजहाँ           

    (d)  औरंगजेब

    उत्तर: (a)

    व्याख्या:

    • भारतीय उपमहाद्वीप के एक विशिष्ट ऐतिहासिक काल के दौरान  मुस्लिम और हिंदू बुद्धिजीवियों ने सह-अस्तित्व, संवाद और एक दूसरे की निकटवर्ती उपस्थिति को समझा।
    • उसी समय अनुवाद टीम के प्रयासों के परिणाम  संस्कृत लघुयोग विसिह को फारसी योग बशिष्ठ के रूप में प्रस्तुत करने से महानगरीय इंडो फारसी दरबारी संस्कृति को महत्त्वपूर्ण योगदान मिला, जो तत्कालीन सम्राट अकबर के प्रोत्साहन के तहत मुगल दरबार में विकसित हुई थी।
    • यह एक ऐसी संस्कृति थी जिसका उद्देश्य मुसलमानों, हिंदुओं और अन्य धार्मिक समूहों के योगदान को एक साथ संश्लेषित करना था।
    • सबसे पहले इन अनुवादों को सम्राट जहाँँगीर द्वारा व्यक्तिगत रूप से अधिकृत किया गया जिसे योग बशिष्ठ के रूप में जाना जाता है, इसे 1597 ईस्वी में तीन सहयोगी अनुवादकों की टीम द्वारा पूरा किया गया था: जिसमे मुस्लिम दरबार के विद्वान निज़ामुद्दीन पानीपति और हिंदू पंडित जगन्नाथ मिश्र बनारसी तथा पठान मिश्रा जाजीपुरी शामिल थे।

    अतः विकल्प (a) सही है।

    https://library.oapen.org/bitstream/id/4018c47c-610c-4597-8692-7dd9c29643a5/translating-wisdom.pdf

    5. हाल ही में हैदराबाद में भारत के प्रधान मंत्री द्वारा रामानुज की आसन मुद्रा में विश्व की दूसरी सबसे ऊँची मूर्ति का उद्घाटन किया गया था। निम्नलिखित कथनों में कौन-सा एक, रामानुज की शिक्षाओं को सही निरूपित करता है?

    (a) मोक्ष प्राप्ति का सर्वोत्तम साधन भक्ति था।

    (b) वेद शाश्वत, आत्म-प्रतिष्ठित तथा पूर्णतया प्रामाणिक हैं।

    (c) तर्कसंगत युक्तियाँ सर्वोच्च आनंद के मौलिक माध्यम थे।

    (d) ध्यान के माध्यम से मोक्ष पाया जा सकता था।

    उत्तर: (a)

    व्याख्या:

    • रामानुज का जन्म ग्यारहवीं शताब्दी में तमिलनाडु में हुआ था। वे विष्णुभक्त अलवार संतो से बहुत प्रभावित थे। उनके अनुसार मोक्ष प्राप्त करने का उपाय विष्णु के प्रति अनन्य भक्ति भाव रखना है। 
      • भगवान विष्णु की कृपा दृष्टि से भक्त उनके साथ एकाकार होने का परमानंद प्राप्त कर सकता है। 
    • रामानुज ने विशिष्टताद्वैत के सिद्धांत को प्रतिपादित किया, जिसके अनुसार आत्मा, परमात्मा से जुड़ने के बाद भी अपनी अलग सत्ता बनाए रखती है।
      • रामानुज के सिद्धांत ने भक्ति की नयी धारा को बहुत प्रेरित किया, जो परवर्ती काल में उत्तरी भारत में विकसित हुई।
    • अतः विकल्प (a) सही है।

    https://ncert.nic.in/ncerts/l/gess108.pdf  

    https://www.drishtiias.com/daily-updates/daily-news-analysis/philosopher-saint-ramanujacharya

    6. हाल ही में, प्रधान मंत्री ने वेरावल में सोमनाथ मंदिर के निकट नए सर्किट हाउस का उद्घाटन किया। सोमनाथ मंदिर के बारे में निम्नलिखित कथनों में कौन-से सही हैं?

    1. सोमनाथ मंदिर ज्योतिर्लिंग देव-मंदिरों में से एक है।
    2. अल-बरूनी ने सोमनाथ मंदिर का वर्णन किया है।
    3. सोमनाथ मंदिर की प्राण-प्रतिष्ठा (आज के मंदिर की स्थापना) राष्ट्रपति एस. राधाकृष्णन द्वारा की गई थी।

    नीचे दिए गए कूट का प्रयोग कर सही उत्तर चुनिए:

    (a) केवल 1 और 2     

    (b)  केवल 2 और 3

    (c) केवल 1 और 3

    (d) 1, 2 और 3

    उत्तर: (a)

    व्याख्या:

    • सोमनाथ मंदिर गुजरात राज्य में भारतीय उपमहाद्वीप के पश्चिमी में अरब सागर के तट पर स्थित है।
    • श्री सोमनाथ भारत के बारह आदि ज्योतिर्लिंगों में प्रथम हैं। अत: कथन 1 सही है।
    • इसका उल्लेख अरब यात्री अल-बरूनी ने अपने यात्रा वृत्तांत में किया था, जिससे प्रभावित होकर महमूद गजनवी ने 1024 ई. में अपने पांँच हज़ार सैनिकों के साथ सोमनाथ मंदिर पर हमला कर दिया और उसकी संपत्ति लूट ली साथ ही मंदिर को पूरी तरह से नष्ट कर दिया। अत: कथन 2 सही है।
    • प्राचीन भारतीय शास्त्रीय ग्रंथों पर आधारित शोध से पता चलता है कि पहली बार सोमनाथ ज्योतिर्लिंग की प्राण-प्रतिष्ठा (आज के मंदिर की स्थापना), वैवस्वत मन्वन्तर के दसवें त्रेता युग के दौरान श्रावण मास की शुक्ल पक्ष की तृतीया तिथि को किया गया था।
    • 13 नवंबर, 1947 को सोमनाथ मंदिर के दर्शन करने वाले सरदार पटेल के संकल्प के साथ आधुनिक मंदिर का पुनर्निर्माण किया गया था। 11 मई, 1951 को भारत के तत्कालीन राष्ट्रपति डॉ. राजेंद्र प्रसाद ने मौजूदा मंदिर में प्राण-प्रतिष्ठा की थी। अतः कथन 3 सही नहीं है।

    https://somnath.org/Home/Somnath-Darshan

    Somnath temple information (indiatourismguide.org)

    7. निम्नलिखित कथनों में कौन-सा एक, मानव शरीर में B कोशिकाओं और ज् कोशिकाओं की भूमिका का सर्वोत्तम वर्णन है?

    (a) वे शरीर को पर्यावरणीय प्रत्यूर्जकों (एलर्जनों) से संरक्षित करती हैं।

    (b) वे शरीर के दर्द और सूजन का अपशमन करती हैं।

    (c) वे शरीर के प्रतिरक्षा-निरोधकों की तरह काम करती हैं।

    (d) वे शरीर को रोगजनकों द्वारा होने वाले रोगों से बचाती हैं।

    उत्तर: (d) 

    व्याख्या:

    • प्राथमिक और द्वितीयक प्रतिरक्षा अनुक्रियाएँ हमारे शरीर के रक्त में मौजूद दो विशेष प्रकार के कोशिकाओं द्वारा होती हैं। ये कोशिकाएँ हैं B-कोशिकाएँ और T-कोशिकाएँ।
    • रोगजनकों की अनुक्रिया में B-कोशिकाएंँ हमारे रक्त में प्रोटीनों की एक शृंखला उत्पन्न करती हैं ताकि वे रोगजनकों से लड़ सकें। ये प्रोटीन प्रतिरक्षी (एंटीबायोटिक्स) कहलाती हैं। T-कोशिकाएँ स्वयं तो प्रतिरक्षियों का श्रावण नहीं करतीं, लेकिन प्रोटीन उत्पन्न करने में B-कोशिकाओं की सहायता करती हैं।
    • अतः विकल्प (d) सही है।

    https://ncert.nic.in/ncerts/l/lebo108.pdf

    https://www.drishtiias.com/daily-updates/daily-news-analysis/covid-19-specific-memory-t-cells

    8. निम्नलिखित कथनों पर विचार कीजिये:

    1. परासूक्ष्मकण (नैनोपार्टिकल्स), मानव-निर्मित होने के सिवाय, प्रकृति में अस्तित्व में नहीं हैं।
    2. कुछ धात्विक ऑक्साइडों के परासूक्ष्मकण, प्रसाधन-सामग्री (कॉस्मेटिक्स) के निर्माण में काम आते हैं।
    3. कुछ वाणिज्यिक उत्पादों के परासूक्ष्मकण, जो पर्यावरण में आ जाते हैं, मनुष्यों के लिए असुरक्षित हैं।

    उपर्युक्त कथनों में कौन-सा/से सही है/हैं?

    (a) केवल 1

    (b)  केवल 3

    (c) 1 और 2

    (d) 2 और 3

    उत्तर: (d)

    व्याख्या:

    • परासूक्ष्मकण (नैनोपार्टिकल्स) पर्यावरण में प्राकृतिक रूप से बड़ी मात्रा में पाए जाते हैं। ये प्रकृति में मौजूद हैं और मानवीय गतिविधियों के परिणामस्वरूप भी निर्मित होते हैं। अतः कथन 1 सही नहीं है। 
    • प्रसाधन उद्योग द्वारा सौंदर्य प्रसाधन, सनस्क्रीन, एंटी-एजिंग क्रीम, मॉइस्चराइज़र और परफ्यूम में सक्रिय घटकों के उचित प्रदर्शन तथा जैव उपलब्धता के लिये नैनोकणों को विकसित कर नैनो तकनीक का लाभ लिया जा रहा है।
    • सौंदर्य प्रसाधन विभिन्न प्रकार के धातु एवं धातु ऑक्साइड नैनोकणों जैसे सिल्वर नैनोपार्टिकल्स (AgNPs), गोल्ड नैनोपार्टिकल्स (AuNPs) एवं टाइटेनियम डाइऑक्साइड नैनोपार्टिकल्स (TiO2 NPs), जिंक ऑक्साइड नैनोपार्टिकल्स, (ZnO NPs), आयरन ऑक्साइड नैनोपार्टिकल्स (Fe2O3 NPs) और कार्बन-आधारित NPs [16–18] का उपयोग करके तैयार किये जाते हैं।
      • टाइटेनियम डाइऑक्साइड (TiO2), जिंक ऑक्साइड (ZnO) जैसे नैनोकणों का अत्यधिक उपयोग किया जाता है, क्योंकि ये नैनोकण गैर-तैलीय होते हैं और आसानी से अवशोषित हो जाते हैं।
      • TiO2 अनिवार्य रूप से यूवी फिल्टर (UVA और UVB फिल्टर) है इसलिये व्यापक रूप से सनस्क्रीन और मॉइस्चराइज़र में भी इसका उपयोग किया जाता है। अत: कथन 2 सही है।
    • मनुष्य प्राकृतिक रूप से पाए जाने वाले अधिकांश नैनोकणों से निपटने में सक्षम है। हालांँकि कुछ नैनोकणों जो मानवीय गतिविधियों जैसे- तंबाकू धूम्रपान और अग्नि के परिणामस्वरूप उत्पन्न होते हैं, फेफड़ों की क्षति के कारण होने वाली मौतों के लिये ज़िम्मेदार हैं। अत: कथन 3 सही है।

    Nanotechnology in cosmetics pros and cons - IOPscience

    nanoparticle - Nanoparticles in the environment। Britannica

    http://surl.li/cdrrj

    9. निम्नलिखित कथनों पर विचार कीजिये:

    DNA बारकोडिंग किसका उपसाधन हो सकता है?

    1. किसी पादप या प्राणी की आयु का आकलन करने के लिए
    2. समान दिखने वाली प्रजातियों के बीच भिन्नता जानने के लिए
    3. प्रसंस्कृत खाद्यपदार्थों में अवांछित प्राणी या पादप सामग्री को पहचानने के लिए

    उपर्युक्त कथनों में कौन-सा/से सही है/हैं?

    (a) केवल 1

    (b)  केवल 3

    (c) 1 और 2

    (d)  2 और 3

    उत्तर: (d)

    व्याख्या: 

    • परमाणु या ऑर्गेनेल जीनोम से लघु DNA अनुक्रमों का उपयोग कर जैविक प्रतिदर्शों की पहचान करने की नई तकनीक को DNA बारकोडिंग कहा जाता है।
    • DNA बारकोडिंग के विभिन्न क्षेत्रों में कई अनुप्रयोग हैं जैसे प्राकृतिक संसाधनों को संरक्षित करना, लुप्तप्राय प्रजातियों की रक्षा करना, कृषि कीटों को नियंत्रित करना, रोग वैक्टर की पहचान करना, पानी की गुणवत्ता की निगरानी करना, प्राकृतिक स्वास्थ्य उत्पादों का प्रमाणीकरण और औषधीय पौधों की पहचान करना।
    • लुप्तप्राय वन्यजीवों की प्रजातियों की पहचान (एक जैसी दिखने वाली प्रजातियों के बीच अंतर), कीट संगरोध और रोग वाहक (अवांछनीय जानवरों / पौधों की पहचान करना) कुछ ऐसे क्षेत्र हैं जिनमें DNA बारकोडिंग शोधकर्त्ताओं, प्रवर्तन एजेंटों और उपभोक्ताओं हेतु बहुत कम समय-सीमा में निर्णय लेती है। अतः कथन 2 और 3 सही हैं, अतः विकल्प (d) सही है।

    https://www.sciencedirect.com/science/article/abs/pii/S0958166913006721 

    http://pnrsolution.org/Datacenter/Vol3/Issue2/291.pdf 

    10. निम्नलिखित पर विचार कीजिये:

    1. कार्बन मोनोक्साइड
    2. नाइट्रोजन ऑक्साइड
    3. ओज़ोन
    4. सल्फर डाइऑक्साइड

    वातावरण में उपर्युक्त में से किसकी/किनकी अधिकता होने से अम्ल वर्षा होती है?

    (a)  1, 2 और 3

    (b)  केवल 2 और 4

    (c) केवल 4

    (d) 1, 3 और 4

    उत्तर: (b)

    व्याख्या:

    • अम्ल वर्षा मानवीय क्रियाकलापों का उपोत्पाद होती है, जो वातावरण में नाइट्रोजन तथा सल्फर के ऑक्साइड निर्गमित करती है। जैसा पूर्व में बताया जा चुका है, जीवाश्म-ईंधन (जैसे- कोयला, शक्ति-संयंत्रें, भट्टियों तथा मोटर इंजनों में डीजल और पेट्रोल, (जिसमें सल्फर तथा नाइट्रोजन पदार्थ होते हैं) के दहन पर सल्फर डाइऑक्साइड तथा नाइट्रोजन ऑक्साइड उत्पन्न होते हैं। 
    • SO2 तथा  NO2  ऑक्सीकरण के पश्चात् जल के साथ अभिक्रिया करके अम्लवर्षा में प्रमुख योगदान देते हैं, क्योंकि प्रदूषित वायु में सामान्यतः कणिकीय द्रव्य उपस्थित होते हैं, जो ऑक्सीकरण को उत्प्रेरित करते हैं।
    • 2SO2 (g) + O2 (g) + 2H2O (l) → 2H2SO4 (aq)
    • 4NO2 (g) + O2 (g)+ 2H2O (l) → 4HNO3 (aq)
    • अतः विकल्प (b) सही है।  

    https://ncert.nic.in/textbook/pdf/kech207.pdf

    https://www.drishtiias.com/daily-updates/daily-news-analysis/sulphur-dioxide-emissions-from-caribbean-volcano/print_manually

    11. निम्नलिखित कथनों पर विचार कीजिये:

    1. उच्च मेघ मुख्यत: सौर विकिरण को परावर्तित कर भूपृष्ठ को ठंडा करते हैं।
    2. भूपृष्ठ से उत्सर्जित होने वाली अवरक्त विकिरणों का निम्न मेघ में उच्च अवशोषण होता है, और इससे तापन प्रभाव होता है।

    उपर्युक्त कथनों में कौन-सा/से सही है/हैं?

    (a) केवल 1 

    (b)  केवल 2

    (c) 1 और 2 दोनों

    (d)  न तो 1, न ही 2

    उत्तर: (d) 

    व्याख्या:

    • बादल जहाँ बनते हैं उसी स्थान पर उनका अध्ययन और उनकी विशेषताएँ, जलवायु परिवर्तन संबंधी ज्ञान में महत्त्वपूर्ण भूमिका निभाती हैं। कम, घने बादल मुख्य रूप से सौर विकिरण को दर्शाते हैं और पृथ्वी की सतह को ठंडा करते हैं। उच्च, पतले बादल मुख्य रूप से आने वाले सौर विकिरण को संचारित करते हैं; साथ ही, वे पृथ्वी द्वारा उत्सर्जित कुछ निवर्तमान अवरक्त विकिरणों में फंस जाते हैं और इसे वापस नीचे की ओर विकीर्ण कर देते हैं, जिससे पृथ्वी की सतह गर्म हो जाती है। अत: दोनों कथन सही नहीं हैं।

    https://earthobservatory.nasa.gov/features/Clouds

    12. निम्नलिखित कथनों पर विचार कीजिये:

    1. उत्तरी-पश्चिमी केन्या में बीड़ीबीड़ी एक वृहद् शरणार्थी बस्ती है।
    2. दक्षिण सूडान गृह युद्ध से पलायन किए हुए कुछ लोग बीड़ीबीड़ी में रहते हैं।
    3. सोमालिया के गृह युद्ध से पलायन किए हुए कुछ लोक केन्या के ददाब शरणार्थी संकुल में रहते हैं।

    उपर्युक्त कथनों में कौन-सा/से सही है/हैं?

    (a) 1 और 2

    (b)  केवल 2

    (c) 2 और 3          

    (d)  केवल 3

    उत्तर: (c)

    व्याख्या:

    • बीड़ीबीड़ी शरणार्थी बस्ती, उत्तर-पश्चिमी युगांडा में एक शरणार्थी शिविर है। अतः कथन 1 सही नहीं है।
    • गृहयुद्ध के कारण 270,000 से अधिक दक्षिण सूडानी शरणार्थी देश से पलायन कर चुके हैं, वर्ष 2017 की शुरुआत में बीड़ीबीड़ी विश्व की सबसे बड़ी शरणार्थी बस्ती थी। अत: कथन 2 सही है।

    https://reliefweb.int/report/uganda/ugandas-bidibidi-refugee-settlement-benefit-iom-and-innovation-norways-electronic-waste-management-project

    • केन्या में ददाब शरणार्थी संकुल में शरणार्थी लोग रहते हैं। इसमें तीन शिविर शामिल हैं। पहला शिविर वर्ष 1991 में स्थापित किया गया था, जब सोमालिया में गृहयुद्ध से भागे शरणार्थियों ने केन्या में सीमा पार करना शुरू किया। वर्ष 2011 में दूसरी अधिकतम शरणार्थी संख्या यहाँ आई जब कुछ शरणार्थी दक्षिणी सोमालिया में सूखे और अकाल के चलते भागकर यहाँ आए। अत: कथन 3 सही है।

    https://www.unhcr.org/ke/dadaab-refugee-complex

    13. निम्नलिखित देशों पर विचार कीजिये:

    1. आर्मीनिया
    2. अज़रबैजान
    3. क्रोएशिया
    4. रोमानिया
    5. उज़्बेकिस्तान

    उपर्युक्त में कौन-से तुर्की राज्यों के संगठन के सदस्य हैं?

    (a)  1, 2 और 4         

    (b) 1 और 3

    (c) 2 और 5          

    (d) 3, 4 और 5

    उत्तर: (C)

    व्याख्या:

    • तुर्की राज्यों के संगठन (तत्कालीन तुर्की भाषी राज्यों की सहयोग परिषद- जिसे तुर्की परिषद कहा जाता है) को वर्ष 2009 में एक अंतर-सरकारी संगठन के रूप में स्थापित किया गया था, जिसका उद्देश्य तुर्क राज्यों के बीच व्यापक सहयोग को बढ़ावा देना था। 
    • इसके चार संस्थापक सदस्य राज्य अजरबैजान, कजाकिस्तान, किर्गिस्तान और तुर्की है। 
    • अक्तूबर 2019 में बाकू में आयोजित 7वें शिखर सम्मेलन के दौरान उज्बेकिस्तान पूर्ण सदस्य के रूप में इसमें शामिल हुआ। 
    • सितंबर 2018 में किर्गिज़ गणराज्य के चोलपोन-अता (Cholpon-Ata) में इसके छठे शिखर सम्मेलन के दौरान हंगरी को संगठन में पर्यवेक्षक का दर्जा प्राप्त हुआ। 
    • हाल ही में नवंबर 2021 में आयोजित 8वें शिखर सम्मेलन में, तुर्कमेनिस्तान संगठन के पर्यवेक्षक सदस्य के रूप में शामिल हुआ। आर्मेनिया, क्रोएशिया और रोमानिया तुर्क राज्यों के संगठन के सदस्य नहीं हैं। अतः विकल्प (c) सही है।

    https://www.turkkon.org/en/turk-konseyi-hakkinda

    14. निम्नलिखित कथनों पर विचार कीजिये:

    1. गुजरात में भारत का विशालतम सौर पार्क है।
    2. केरल में पूर्णत: सौर शक्तिकृत अंतर्राष्ट्रीय हवाई अड्डा है।
    3. गोआ में भारत की विशालतम तैरती हुई सौर प्रकाश-वोल्टीय परियोजना है।

    उपर्युक्त कथनों में कौन-सा/से सही है/हैं?

    (a) 1 और 2

    (b)  केवल 2

    (c) 1 और 3

    (d) केवल 3

    उत्तर: (b)

    व्याख्या:

    • भारत का भादला सोलर पार्क विश्व का सबसे बड़ा सोलर पॉवर पार्क है। भादला सोलर पार्क राजस्थान के सूखे तथा रेतीले क्षेत्र में स्थित है और 14,000 एकड़ क्षेत्रफल में फैला हुआ है। पार्क में 10 मिलियन से अधिक सौर पैनल हैं, जो 2245MW विद्युत उत्पादन क्षमता के उत्पादन योगदान देते हैं। अतः कथन 1 सही नहीं है।

    https://www.thehindu.com/sci-tech/energy-and-environment/worlds-largest-solar-park-in-bhadla-india/article37462665.ece

    • केरल का कोचीन इंटरनेशनल एयरपोर्ट लिमिटेड (CIAL) विश्व का पहला हवाई अड्डा है जो पूरी तरह से सौर ऊर्जा संचालित होगा। हवाई अड्डे ने वर्ष 2015 में आधिकारिक तौर पर 12 मेगावाट की सौर परियोजना शुरू की थी। अतः कथन 2 सही है।

    https://www.icao.int/environmental-protection/Documents/EnvironmentalReports/2016/ENVReport2016_pg177-177.pdf

    • तेलंगाना के पेद्दापल्ली ज़िले के रामागुंडम में नेशनल थर्मल पावर कॉरपोरेशन लिमिटेड (NTPC) द्वारा भारत का सबसे बड़ा 100MW उत्पादन क्षमता वाला तैरता सौर ऊर्जा संयंत्र विकसित किया जा रहा है। अत: कथन 3 सही नहीं है।

    https://www.thehindu.com/news/national/telangana/indias-biggest-floating-solar-plant-to-be-commissioned-in-next-three-months/article34036031.ece

    https://www.drishtiias.com/state-pcs-current-affairs/two-new-solar-parks-in-jaisalmer-and-bikaner

    https://www.drishtiias.com/daily-updates/daily-news-analysis/india-s-biggest-floating-solar-power-plant

    15. समुद्री कानून पर संयुक्त राष्ट्र अभिसमय (यूनाइटेड नेशंस कन्वेंशन) के सन्दर्भ में, निम्नलिखित कथनों पर विचार कीजिये:

    1. किसी तटीय राज्य को, अपने प्रादेशिक समुद्र की चौड़ाई को, आधार-रेखा से मापित, 12 समुद्री मील से अनधिक सीमा तक अभिसमय के अनुरूप सुस्थापित करने का अधिकार है।
    2. सभी राज्यों के, चाहे वे तटीय हों या भू-बद्ध भाग के हों, जहाजों को प्रादेशिक समुद्र से हो कर बिना रोक-टोक यात्रा का अधिकार होता है।
    3. अनन्य आर्थिक क्षेत्र का विस्तार उस आधार-रेखा से 200 समुद्री मील से अधिक नहीं होगा, जहाँ से प्रादेशिक समुद्र की चौड़ाई मापी जाती है।

    उपर्युक्त कथनों में कौन-से सही हैं?

    (a) केवल 1 और 2

    (b)  केवल 2 और 3

    (c) केवल 1 और 3

    (d) 1, 2 और 3

    उत्तर: (d)

    व्याख्या:

    • प्रादेशिक समुद्र की सीमा के तहत प्रत्येक राज्य को अपने प्रादेशिक समुद्र के विस्तार को 12 समुद्री मील से अधिक की सीमा तक स्थापित करने का अधिकार है, जिसे इस अभिसमय के अनुसार निर्धारित आधार रेखा से मापा जाता है। अत: कथन 1 सही है।
    • प्रादेशिक समुद्र में इनोसेंट पैसेज के तहत सभी राज्यों (चाहे वे तटीय हों या स्थलबद्ध) के जहाज़ों को इस अभिसमय के तहत प्रादेशिक समुद्र में मार्ग का अधिकार है। अत: कथन 2 सही है।
    • इस अभिसमय के अनुसार, अनन्य आर्थिक क्षेत्र इस भाग में स्थापित विशिष्ट कानूनी व्यवस्था के अधीन प्रादेशिक समुद्र से परे और उससे सटे क्षेत्र हैं, जिसके तहत तटीय राज्य के अधिकार क्षेत्र और अन्य राज्यों के अधिकार एवं स्वतंत्रता प्रासंगिक प्रावधानों द्वारा शासित होते हैं।
    • इसके तहत अनन्य आर्थिक क्षेत्र उन बेसलाइनों से 200 समुद्री मील से अधिक नहीं विस्तारित होगा जहाँ से प्रादेशिक समुद्र की चौड़ाई मापी जाती है। अत: कथन 3 सही है।

    https://www.un.org/depts/los/convention_agreements/texts/unclos/unclos_e.pdf

    https://www.drishtiias.com/daily-updates/daily-news-analysis/unclos-1

    16. निम्नलिखित कथनों में कौन-सा एक, कभी-कभी समाचारों में उल्लिखित सेंकाकू द्वीप विवाद को सर्वोत्तम रूप में प्रतिबिंबित करता है?

    (a) आम तौर पर यह माना जाता है कि वे दक्षिणी चीन सागर के आसपास किसी देश द्वारा निर्मित कृत्रिम द्वीप हैं।

    (b) चीन और जापान के बीच पूर्वी चीन सागर में इन द्वीपों के विषय में समुद्री विवाद होता रहता है।

    (c) वहाँ ताइवान को अपनी रक्षा क्षमताओं को बढ़ाने में मदद करने के लिए एक स्थायी अमेरिकी सैन्य अड्डा स्थापित किया गया है।

    (d) यद्यपि अंतर्राष्ट्रीय न्यायालय ने उन्हें अस्वामिक भूमि घोषित किया है, तथापि कुछ दक्षिण-पूर्वी एशियाई देश उन पर दावा करते हैं।

    उत्तर: (b)

    व्याख्या:

    • सेनकाकू द्वीप कृत्रिम द्वीप नहीं बल्कि एक प्राकृतिक द्वीप है। यह आठ निर्जन द्वीपों का समूह है। अतः कथन 1 सही नहीं है।
    • सेनकाकू द्वीप विवाद चीन और जापान के बीच ज्ञात निर्जन द्वीपों के एक समूह पर क्षेत्रीय विवाद से संबंधित है। जापान और चीन दोनों ही इन द्वीपों के स्वामित्व का दावा करते हैं। सेनकाकू द्वीप पूर्वी चीन सागर में स्थित है। अत: कथन 2 सही है।
    • अमेरिका ने सेनकाकू द्वीप पर स्थायी सैन्य ठिकाने स्थापित नहीं किये लेकिन अमेरिका और जापान सैन्य बलों के बीच संयुक्त सैन्य अभ्यास किया गया है। अत: कथन 3 सही नहीं है।
    • अंतर्राष्ट्रीय न्यायालय ने सेनकाकू द्वीप के संबंध में नो मैन्स लैंड जैसा कोई फैसला नहीं दिया है। अतः कथन 4 सही नहीं है।

    https://www.drishtiias.com/daily-updates/daily-news-analysis/senkaku-island-dispute

    17. निम्नलिखित युग्मों पर विचार कीजिये:

    देश

    हाल ही में समाचारों में होने का महत्त्वपूर्ण कारण

    1. चाड

    चीन द्वारा स्थायी सैन्य बेस की स्थापना

    2. गिनी

    सेना द्वारा संविधान और सरकार का निलंबन

    3. लेबनान

    गंभीर और लंबे समय की आर्थिक मंदी

    4. ट्यूनीशिया

    राष्ट्रपति द्वारा संसद का निलंबन

    उपर्युक्त युग्मों में कितने सही सुमेलित हैं?

    (a) केवल एक युग्म     

    (b) केवल दो युग्म

    (c) केवल तीन युग्म    

    (d) सभी चारों युग्म

    उत्तर. C

    व्याख्या:

    • गिनी में सैन्य नेताओं ने राष्ट्रपति को हिरासत में लिया और सरकार को भंग करने तथा संविधान को निलंबित करने की घोषणा की।

    https://www.thehindu.com/news/international/soldiers-detain-guineas-president-dissolve-government/article36307866.ece 

    लेबनान: गंभीर और दीर्घकालीन आर्थिक मंदी

    • लेबनान एक आयात-निर्भर देश है। बुरी तरह क्षतिग्रस्त पोर्ट फैसिलिटी लेबनान का सबसे बड़ा समुद्री प्रवेश द्वार है तथा यह आवश्यक वस्तुओं को महंगा कर देगा तथा देश में खाद्य सुरक्षा के लिये खतरा उत्पन्न हो जाएगा।
    • लेबनान पहले से ही स्थानीय मुद्रा के तेजी से अवमूल्यन और मुद्रास्फीति, बंद होते व्यवसायों, बेरोज़गारी और गरीबी को बढ़ावा देने वाले अस्थिर विनिमय दर के साथ एक बड़ी आर्थिक मंदी से जूझ रहा है।
    • इसने मार्च 2020 में यूरोबॉण्ड के पुनर्भुगतान में भी चूक की थी।
    • ट्यूनीशियाई राष्ट्रपति ने प्रधानमंत्री को निलंबित कर दिया और संसद भंग कर दी है।

    https://indianexpress.com/article/explained/explained-why-did-the-tunisian-president-suspend-the-prime-minister-and-dissolve-parliament-7438977/ 

    अतः विकल्प (c) सही है।

    18. निम्नलिखित युग्मों पर विचार कीजिये:

    अक्सर समाचारों में उल्लिखित क्षेत्र देश
    1. अनातोलिया तुर्की
    2. अम्हारा इथियोपिया
    3. काबो डेलगादो स्पेन
    4. कातालोनिया इटली

    उपर्युक्त युग्मों में से कितने सही सुमेलित हैं?

    (a) केवल एक युग्म     

    (b)  केवल दो युग्म

    (c) केवल तीन युग्म    

    (d)  सभी चारों युग्म

    उत्तर: (b)

    व्याख्या:

    • अनातोलिया - तुर्की
    • अमहारा- इथियोपिया

    https://www.drishtiias.com/daily-updates/daily-news-analysis/conflict-in-ethiopia 

    • काबो डेलगाडो- मोज़ाम्बिक
    • कैटेलोनिया-  स्पेन

    https://www.drishtiias.com/daily-updates/daily-news-analysis/catalonian-unrest 

    अत: विकल्प b सही है।

    19. वन्यजीव संरक्षण के बारे में भारतीय विधियों के सन्दर्भ में, निम्नलिखित कथनों पर विचार कीजिये:

    1. वन्यजीव, एकमात्र सरकार की संपत्ति हैं।
    2. जब किसी वन्यजीव को संरक्षित घोषित किया जाता है, तो यह जीव चाहे संरक्षित क्षेत्र में हो या उससे बाहर, समान संरक्षण का हकदार है।
    3. किसी संरक्षित वन्यजीव के मानव जीवन के लिए खतरा बन जाने की आश्ंाका उस जीव को पकड़ने या मार दिए जाने का पर्याप्त आधार है।

    उपर्युक्त कथनों में कौन-सा/से सही है/हैं?

    (a) 1 और 2          

    (b) केवल 2

    (c) 1 और 3

    (d)  केवल 3

    उत्तर: (a)

    व्याख्या:

    • एक महत्त्वपूर्ण फैसले में बॉम्बे उच्च न्यायालय ने फैसला सुनाया है कि बाघों सहित जंगली जानवरों को "सभी उद्देश्यों के लिये सरकारी संपत्ति" माना जाना चाहिये और उनके द्वारा किये गए किसी भी नुकसान की भरपाई सरकार द्वारा की जानी चाहिये। अतः कथन 1 सही है।

    http://archive.indianexpress.com/news/now-all-wild-animals-are-govt-property/936369/ 

    • जब किसी जानवर को वन्यजीव (संरक्षण) अधिनियम (WPA), 1972 के तहत संरक्षित जानवर घोषित किया जाता है, तो उसे उसी सुरक्षा का लाभ मिलेगा, चाहे वह संरक्षित क्षेत्र में मौजूद हो या संरक्षित क्षेत्र के बाहर। अत: कथन 2 सही है।
    • वन्यजीव संरक्षण अधिनियम, 1972 की धारा 11 (1) (ए) के अनुसार, यदि मुख्य वन्यजीव वार्डन संतुष्ट है कि अनुसूची 1 में निर्दिष्ट कोई जंगली जानवर मानव जीवन के लिये खतरनाक हो गया है या इतना विकलांग या मृतप्राय है कि ठीक नही हो सकता तो ऐसे ऐसे में लिखित आदेश द्वारा और उसका कारण बताते हुए, किसी भी व्यक्ति को ऐसे जानवर का शिकार करने की अनुमति देता है। बशर्ते किसी भी जंगली जानवर को मारने का आदेश तब तक नहीं दिया जाएगा जब तक कि मुख्य वन्यजीव वार्डन संतुष्ट न हो जाए कि ऐसे जानवर को पकड़ा, शांत या स्थानांतरित नहीं किया जा सकता है। ऐसे जानवर का जंगल में पुनर्वास नहीं किये जा सकने के कारणों को लिखित रूप में दर्ज किया जाता है। स्पष्टीकरण - खंड (ए) के प्रयोजनों के लिये इस तरह के एक जानवर को पकड़ने या स्थानांतरित करने की प्रक्रिया जैसा भी मामला हो इस तरह से किया जाएगा कि उक्त जानवर को न्यूनतम आघात पहुँचे।] अतः कथन 3 सही नहीं है।

    https://legislative.gov.in/sites/default/files/A1972-53_0.pdf 

    https://www.drishtiias.com/to-the-points/paper3/wildlife-protection-act-wpa-1972

    20. निम्नलिखित में से किस एक जीव की कुुछ प्रजातियाँ कवकों के कृषकों के रूप में जानी जाती हैं?

    (a) चींटी 

    (b)  कॉक्रोच

    (c) केकड़ा 

    (d)  मकड़ी

    उत्तर: (a)

    व्याख्या: 

    • चींटी की प्रजाति को कवक के कृषकों के रूप में जाना जाता है। कुछ अन्य कीट जैसे दीमक, भृंग और मार्श पेरिविंकल भी कवक के कृषकों के रूप में जाने जाते हैं।

    अत: विकल्प A सही है।

    https://www.sciencedirect.com/science/article/pii/S1367593120301095

    21. भारत में, निम्नलिखित में कौन एक, उन पैक्टरियों में जिनमें कामगार नियुक्त हैं, औद्योगिक विवादों, समापनों, छँटनी और कामबंदी के विषय में सूचनाओं को संकलित करता है?

    (a) केन्द्रीय सांख्यिकी कार्यालय                

    (b) उद्योग संवर्धन और आंतरिक व्यापार विभाग

    (c) श्रम ब्यूरो

    (d) राष्ट्रीय तकनीकी जनशक्ति सूचना प्रणाली

    उत्तर:(c)

    व्याख्या:

    • भारत में औद्योगिक विवादों, बंदियों, छंटनी और छंटनी पर सांख्यिकी श्रम ब्यूरो का एक वार्षिक प्रकाशन है जो श्रम एवं रोज़गार मंत्रालय के तहत एक संलग्न कार्यालय है। अतः विकल्प (c) सही है।

    श्रम ब्यूरो:

    • श्रम और रोज़गार मंत्रालय के तहत एक संलग्न कार्यालय के रूप में श्रम ब्यूरो की स्थापना 1 अक्तूबर, 1946 को की  गई थी।
    • श्रम ब्यूरो मज़दूरी, कमाई, उत्पादकता, अनुपस्थिति, श्रम कारोबार, औद्योगिक संबंध, काम करने और रहने की स्थिति तथा विभिन्न श्रम अधिनियमों के कामकाज के मूल्यांकन आदि के संबंध में आँकड़े एवं संबंधितजानकारी एकत्रित व प्रकाशित करता है। उद्योगों के लिये उपभोक्ता मूल्य सूचकांक संख्या जैसे महत्त्वपूर्ण आर्थिक संकेतकों के अलावा, कृषि और ग्रामीण मज़दूर; मज़दूरी दर सूचकांक और औद्योगिक संबंधों पर आंँकड़े, संगठित और असंगठित क्षेत्र में उद्योगों की सामाजिक-आर्थिक स्थिति आदि भी कार्यालय द्वारा जारी किये जाते हैं।

    Labour Bureau - Arthapedia

    ID_Review_2012.pdf (labourbureaunew.gov.in)

    22. भारत में, कोयला नियंत्रक संगठन (Coal Controller's Organization – CCO) की क्या भूमिका है?

    1. CCO भारत सरकार में कोयला सांख्यिकी का प्रमुख स्रोत है।
    2. यह बद्ध कायेला/लिग्नाइट खंड के विकास की प्रगति का मॉनीटरन करता है।
    3. यह कोयलायुक्त क्षेत्रों के अधिग्रहण के संबंध में सरकार की अधिसूचना के प्रति किसी आपत्ति का अनुश्रवण करता है।
    4. यह सुनिश्चित करता है कि कोयला खनन कंपनियाँ विहित समय में अंतिम उपभोक्ताओं को कोयला वितरण करें।

    नीचे दिए कूट का प्रयोग कर सही उत्तर चुनिए:

    (a) 1, 2 और 3      

    (b)  केवल 3 और 4

    (c) केवल 1 और 2

    (d)  1, 2 और 4

    उत्तर: (a)

    कोयला नियंत्रक संगठन:

    • वर्ष 1916 में स्थापित कोयला नियंत्रक का कार्यालय (पहले कोयला आयुक्त), भारतीय कोयला क्षेत्र के सबसे पुराने कार्यालयों में से एक है। इस कार्यालय की स्थापना का मुख्य उद्देश्य प्रथम विश्व युद्ध के दौरान कोयले की आवश्यकता की पूर्ति पर सरकारी नियंत्रण रखना था।

    कोयला नियंत्रक संगठन के कार्य नीचे सूचीबद्ध हैं:

    • कोयले की श्रेणी, ग्रेड या आकार की शुद्धता सुनिश्चित करने के लिये कोयला-खानों का निरीक्षण। कोयला खान (colliery) में खनन किये गए सीम के कोयले के ग्रेड की घोषणा और रखरखाव के उद्देश्य से निर्देश जारी करना।
    • केंद्र और राज्य सरकार के विभिन्न मंत्रालयों, राष्ट्रीय और अंतर्राष्ट्रीय संगठनों को मासिक कोयला डेटा प्रस्तुत करना। अत: कथन 1 सही है।
    • कोयला युक्त क्षेत्र (अधिग्रहण एवं विकास) अधिनियम, 1957 के तहत कोयला नियंत्रक इस अधिनियम के तहत सक्षम प्राधिकारी है जो कोयला आधारित भूमि के अधिग्रहण से संबंधित केंद्र सरकार की अधिसूचना पर किसी भी आपत्ति को सुनने और केंद्र सरकार को अपनी रिपोर्ट प्रस्तुत करने हेतु सक्षम है। अत: कथन 3 सही है।
    • कोयला नियंत्रक संगठन कैप्टिव कोयला/लिग्नाइट ब्लॉकों के विकास और उनकी संबद्ध अंतिम-उपयोग परियोजनाओं की प्रगति की निगरानी करता है। अत: कथन 2 सही है।
    • यह सुनिश्चित नहीं करता है कि कोयला खनन कंपनी निर्धारित समय में अंतिम उपयोगकर्त्ता तक कोयला पहुँचाती है। अत: कथन 4 सही नहीं है।

    http://coalcontroller.gov.in/pages/display/5-functionsresponsibilities#:~:text=Under%20Coal%20Bearing%20Area%20%28Acquisition%20and%20Deve

    http://surl.li/cdrxl

    23. यदि किसी विशिष्ट क्षेत्र को भारत के संविधान की पाँचवी अनुसूची के अधीन लाया जाए, तो निम्नलिखित कथनों में कौन-सा एक, इसके परिणाम को सर्वोत्तम रूप से प्रतिबिंबित करता है?

    (a) इससे जनजातीय लोगों की जमीनें गैर-जनजातीय लोगों को अंतरित करने पर रोक लगेगी।

    (b) इससे उस क्षेत्र में एक स्थानीय स्वशासी निकाय का सृजन होगा।

    (c) इससे वह क्षेत्र संघ राज्यक्षेत्र में बदल जाएगा।

    (d) जिस राज्य के पास ऐसे क्षेत्र होंगे, उसे विशेष कोटि का राज्य घोषित किया जाएगा।

    उत्तर: (a) 

    पांचवीं अनुसूची क्षेत्र के अंतर्गत भूमि शासन:

    • राज्यपाल निम्नलिखित के संबंध में नियम बना सकता है:

    (i) अनुसूचित जनजातियों से और उनके बीच भूमि के हस्तांतरण का निषेध और प्रतिबंध – देश के लगभग हर राज्य और निश्चित रूप से अनुसूचित क्षेत्रों वाले सभी राज्यों ने अनुसूचित क्षेत्रों में आदिवासियों द्वारा गैर-आदिवासियों को भूमि हस्तांतरण की रोकथाम/निषेध से संबंधित कानून बनाए हैं और कुछ मामलों में, आदिवासियों के बीच भूमि का परस्पर हस्तांतरण तक प्रतिबंधित है। अतः विकल्प (a) सही है।

    (ii) अनुसूचित क्षेत्रों में आदिवासियों को भूमि आवंटन का विनियमन;

    (iii)अनुसूचित क्षेत्रों में आदिवासियों को साहूकारी का विनियमन।

    https://www.mea.gov.in/Images/pdf1/S5.pdf 

    https://tribal.nic.in/downloads/FRA/5.%20Land%20and%20Governance%20under%20Fifth%20Schedule.pdf

    24. निम्नलिखित कथनों पर विचार कीजिये:

    1. भारत स्वच्छता गठबंधन धारणीय स्वच्छता को संवर्धित करने वाला प्लेटफॉर्म है और भारत सरकार तथा विश्व स्वास्थ्य संगठन द्वारा इसका वित्तपोषण होता है।
    2. राष्ट्रीय नगर कार्य संस्थान भारत सरकार में आवासन एवं शहरी कार्य मंत्रालय का शीर्षस्थ निकाय है, और यह शहरी भारत की चुनौतियों का समाधान करने के नवप्रवर्तक हल उपलब्ध कराता है।

    उपर्युक्त कथनों में कौन-सा/से सही है/हैं?

    (a) केवल 1

    (b)  केवल 2

    (c) 1 और 2 दोनों  

    (d)  न तो 1, न ही 2

    उत्तर: (b)

    भारत स्वच्छता गठबंधन (India Sanitation Coalition)

    • भारत स्वच्छता गठबंधन का गठन 'फिक्की के तत्त्वावधान में किया गया था।
    • गठबंधन का दृष्टिकोण एक साझेदारी मोड की सहायता से धारणीय स्वच्छता के लिये एक पारिस्थितिकी तंत्र को सक्षम और समर्थन करना है। अतः कथन 1 सही नहीं है।

    शहरी मामलों का राष्ट्रीय संस्थान

    • वर्ष 1976 में स्थापित, शहरी मामलों का राष्ट्रीय संस्थान/नेशनल इंस्टीट्यूट ऑफ अर्बन अफेयर्स (NIUA) शहरी नियोजन और विकास पर भारत का प्रमुख राष्ट्रीय विचार मंच है।
    • शहरी क्षेत्र में अत्याधुनिक अनुसंधान के निर्माण और प्रसार के केंद्र के रूप में, NIUA तेज़ी से शहरीकरण के क्रम में भारत की चुनौतियों का समाधान करने के लिये अभिनव समाधान प्रदान करने का उद्देश्य रखता है और भविष्य के अधिक समावेशी तथा संवहनीय शहरों का मार्ग प्रशस्त करता है।
    • वर्ष 1976 में, NIUA को भारत सरकार की शहरी विकास योजनाओं में समर्थन और मार्गदर्शन करने के लिटे एक शीर्ष निकाय के रूप में नियुक्त किया गया था। तब से, इसने आवास और शहरी मामलों के मंत्रालय के साथ- साथ अन्य सरकारी और नागरिक क्षेत्रों के साथ मिलकर, अनुसंधान के प्रमुख क्षेत्रों की पहचान करने तथा शहरी नीति एवं नियोजन में कमियों को दूर करने का काम किया है। अतः कथन 2 सही है।

    Ficci-press-jun25-sanitation.pdf

    Who We Are - India Sanitation Coalition

    Sustainable Cities India Program (drishtiias.com)

    25. निम्नलिखित में कौन-सा एक, पर्यावरण (संरक्षण) अधिनियम, 1986 के अधीन गठित किया गया है?

    (a) केन्द्रीय जल आयोग

    (b) केन्द्रीय भूजल बोर्ड

    (c) केन्द्रीय भूजल प्राधिकरण

    (d) राष्ट्रीय जल विकास अभिकरण

    उत्तर: (c)

    • देश में भूजल संसाधनों के विकास और प्रबंधन को विनियमित एवं नियंत्रित करने के लिये पर्यावरण (संरक्षण) अधिनियम, 1986 की धारा 3 (3) के तहत केंद्रीय भूजल प्राधिकरण का गठन किया गया है। अतः विकल्प (c) सही है।

    शक्तियाँ एवं कार्य:

    प्राधिकरण को निम्नलिखित शक्तियाँ प्रदान की गई हैं:

    (i) उक्त अधिनियम की धारा 3 की उप-धारा (2) में निर्दिष्ट सभी मामलों के संबंध में निर्देश जारी करने और ऐसे उपाय करने के लिये पर्यावरण (संरक्षण) अधिनियम, 1986 की धारा 5 के तहत शक्तियों का प्रयोग।

    (ii)  उक्त अधिनियम की धारा 15 से 21 में निहित दंडात्मक प्रावधानों की सहायता लेना।

    (iii)  देश में भूजल का विनियमन और नियंत्रण, प्रबंधन एवं विकास करना तथा इस उद्देश्य के लिये आवश्यक नियामक निर्देश जारी करना।

    (iv)  अधिकारियों की नियुक्ति के लिये पर्यावरण (संरक्षण) अधिनियम, 1986 की धारा 4 के तहत शक्तियों का प्रयोग।

    INDEX (cgwb.gov.in)

    26. ‘‘संयुक्त राष्ट्र प्रत्यय समिति (युनाईटेड नेशंस क्रेडेंशियल्स कमिटी)’’ के सन्दर्भ में, निम्नलिखित कथनों पर विचार कीजिये:

    1. यह संयुक्त राष्ट्र (UN) सुरक्षा परिषद् द्वारा स्थापित समिति है और इसके पर्यवेक्षण के अधीन काम करती है।
    2. पारंपरिक रूप से प्रति वर्ष मार्च, जून और सितंबर में इसकी बैठक होती है।
    3. यह महासभा को अनुमोदन हेतु रिपोर्ट प्रस्तुत करने से पूर्व सभी UN सदस्यों के प्रत्ययों का आकलन करती है।

    उपर्युक्त कथनों में कौन-सा/से सही है/हैं?

    (a) केवल 3

    (b) 1 और 3

    (c) 2 और 3

    (d) 1 और 2

    उत्तर: (A)

    • संयुक्त राष्ट्र महासभा (UNGA) के प्रत्येक नियमित सत्र की शुरुआत में एक प्रत्यय समिति नियुक्त की जाती है।
    • इसमें नौ सदस्य होते हैं, जिन्हें UNGA अध्यक्ष के प्रस्ताव पर महासभा द्वारा नियुक्त किया जाता है। अतः कथन 1 सही नहीं है।
    • समिति प्रतिनिधियों की साख पर विधानसभा को रिपोर्ट करती है।
    • प्रतिनिधियों की साख और प्रत्येक सदस्य राज्य के प्रतिनिधिमंडल के सदस्यों के नाम महासचिव को प्रस्तुत किये जाते हैं और या तो राज्य या सरकार के प्रमुख या विदेश मामलों के मंत्री द्वारा जारी किये जाते हैं।
    • समिति के लिये सदस्य राज्यों के प्रतिनिधियों की साख की जाँच करना और उस पर महासभा को रिपोर्ट करना अनिवार्य है। अत: कथन 3 सही है।
    • आमतौर पर, समिति की बैठक नवंबर में होती है, दिसंबर में आम सभा के समक्ष रिपोर्ट प्रस्तुत की जाती है। अतः कथन 2 सही नहीं है।

    https://www.un.org/en/ga/credentials/credentials.shtml 

    https://theprint.in/world/un-credentials-committee-that-will-review-taliban-nomination-likely-to-meet-in-november/744076/ 

    27. निम्नलिखित में से कौन-सा एक कथन ‘ध्रुवीय कोड (Polar Code)’ का सर्वोत्तम वर्णन करता है?

    (a) ध्रुवीय जलराशियों में परिचालन कर रहे जहाज़ों के लिए यह सुरक्षा का अंतर्राष्ट्रीय कोड है।

    (b) यह उत्तरी ध्रुव के आसपास के देशों का ध्रुवीय क्षेत्र में अपने राज्यक्षेत्रों के सीमांकन का समझौता है।

    (c) यह उत्तरी ध्रुव और दक्षिणी ध्रुव में अनुसंधान करने वाले वैज्ञानिकों के देशों द्वारा अपनाए जाने वाले मानकों का समुच्चय है।

    (d) यह आर्कटिक कौंसिल के सदस्य देशों का व्यापारिक और सुरक्षा समझौता है।

    उत्तर: (A)

    • ध्रुवीय जल में संचालित जहाज़ों के लिये IMO का अंतर्राष्ट्रीय कोड (ध्रुवीय कोड) समुद्र में जीवन की सुरक्षा के लिये अंतर्राष्ट्रीय सम्मेलन (SOLAS) और जहाज़ों से प्रदूषण की रोकथाम के लिये अंतर्राष्ट्रीय सम्मेलन (MARPOL) दोनों के तहत अनिवार्य है। ध्रुवीय संहिता में डिज़ाइन, निर्माण, उपकरण, परिचालन, प्रशिक्षण, खोज और बचाव तथा दो ध्रुवों के आस-पास के दुर्गम जल में चलने वाले जहाज़ों के लिये प्रासंगिक पर्यावरण संरक्षण मामलों की पूरी शृंखला शामिल है। अतः विकल्प (a) सही है।
    • ध्रुवीय संहिता 1 जनवरी 2017 को लागू हुई।

    https://www.imo.org/en/OurWork/Safety/Pages/polar-code.aspx

    28. संयुक्त राष्ट्र महासभा के सन्दर्भ में, निम्नलिखित कथनों पर विचार कीजिये:

    1. UN महासभा, गैर-सदस्य राज्यों को प्रेक्षक स्थिति प्रदान कर सकती है।
    2. अंत:सरकारी संगठन UN महासभा में प्रेक्षक स्थिति पाने का प्रयत्न कर सकते हैं।
    3. UN महासभा में स्थायी प्रेक्षक UN मुख्यालय में मिशन बनाए रख सकते हैं।

    उपर्युक्त कथनों में कौन-से सही हैं?

    (a) केवल 1 और 2

    (b)  केवल 2 और 3

    (c) केवल 1 और 3

    (d)  1, 2 और 3

    उत्तर: (D)

    • संयुक्त राष्ट्र के गैर-सदस्य राज्य, जो एक या अधिक विशिष्ट एजेंसियों के सदस्य हैं, स्थायी पर्यवेक्षक के दर्जे के लिये आवेदन कर सकते हैं।
    • संयुक्त राष्ट्र महासभा गैर-सदस्य राज्यों, अंतर्राष्ट्रीय संगठनों और अन्य संस्थाओं को स्थायी पर्यवेक्षक का दर्जा दे सकती है। अतः कथन 1 और 2 सही हैं।
    • एक स्थायी पर्यवेक्षक का दर्जा विशुद्ध रूप से अभ्यास पर आधारित होता है और संयुक्त राष्ट्र चार्टर में इसके लिये कोई प्रावधान नहीं हैं।
    • इस प्रणाली की शुरुआत वर्ष 1946 में हुई, जब महासचिव ने संयुक्त राष्ट्र में एक स्थायी पर्यवेक्षक के रूप में स्विस सरकार के पद को स्वीकार किया।
    • धीरे-धीरे कुछ राज्यों द्वारा पर्यवेक्षकों को आगे रखा गया जो बाद में संयुक्त राष्ट्र के सदस्य बन गए इनमें ऑस्ट्रिया, फिनलैंड, इटली और जापान शामिल थे। 10 सितंबर, 2002 को स्विट्ज़रलैंड संयुक्त राष्ट्र का सदस्य बना।
    • स्थायी पर्यवेक्षकों के पास अधिकांश बैठकों और प्रासंगिक दस्तावेज़ों तक निशुल्क पहुँच होती है।
    • कई क्षेत्रीय और अंतर्राष्ट्रीय संगठन भी महासभा के कार्य और वार्षिक सत्रों में पर्यवेक्षक हैं।
    • स्थायी पर्यवेक्षक संयुक्त राष्ट्र मुख्यालय में महासभा के सत्रों और कार्यों में भाग ले सकते हैं और मिशनों को जारी रख सकते हैं। अतः कथन 3 सही है।

    https://www.un.org/en/about-us/about-permanent-observers

    https://ask.un.org/faq/14519

    29. भारत में ‘‘चाय बोर्ड’’ के सन्दर्भ में, निम्नलिखित कथनों पर विचार कीजिये:

    1. चाय बोर्ड सांविधिक निकाय है।
    2. यह कृषि एवं किसान कल्याण मंत्रालय से संलग्न नियामक निकाय है।
    3. चाय बोर्ड का प्रधान कार्यालय बेंगलुरु में स्थित है।
    4. इस बोर्ड के दुबई और मॉस्को में विदेशी कार्यालय हैं।

    उपर्युक्त कथनों में कौन-से सही हैं?

    (a) 1 और 3

    (b) 2 और 4

    (c) 3 और 4          

    (d) 1 और 4

    उत्तर: (D)

    • चाय बोर्ड केंद्र सरकार के एक सांविधिक निकाय के रूप में कार्य कर रहा है। अतः कथन 1 सही है।
    • यह वाणिज्य मंत्रालय के अंतर्गत आता है। अतः कथन 2 सही नहीं है।
    • बोर्ड का गठन 31 सदस्यों (अध्यक्ष सहित) से होता है, जो संसद सदस्यों, चाय उत्पादकों, चाय व्यापारियों, चाय दलालों, उपभोक्ताओं और प्रमुख चाय उत्पादक राज्यों की सरकारों के प्रतिनिधियों और ट्रेड यूनियनों में से चुने जाते हैं। प्रत्येक तीन वर्ष में बोर्ड का पुनर्गठन किया जाता है।
    • विदेशों में कार्यालय: वर्तमान में चाय बोर्ड के दो विदेशी कार्यालय (दुबई और मॉस्को में) हैं। बोर्ड के इन सभी विदेशी कार्यालयों को भारतीय चाय के निर्यात को बढ़ावा देने हेतु विभिन्न प्रचार उपायों के लिये डिज़ाइन किया गया है। ये कार्यालय संबंधित क्षेत्रों में भारतीय चाय के आयातकों के साथ-साथ भारतीय निर्यातकों के बीच बातचीत के लिये एक संपर्क कार्यालय के रूप में भी कार्य करते हैं। अतः कथन 4 सही है।
    • इसका मुख्यालय कोलकाता में स्थित है। अतः कथन 3 सही नहीं है।

    https://www.teaboard.gov.in/TEABOARDCSM/NA==

    30. निम्नलिखित में कौन-सा एक, ‘‘ग्रीनवाशिंग’’ शब्द का सर्वोत्तम वर्णन है?

    (a) मिथ्या रूप से यह प्रभाव व्यक्त करना कि कंपनी के उत्पाद पारिस्थितिक-अनुकूली (ईको-फ्रेंडली) और पर्यावरणीय रूप से उपयुक्त हैं

    (b) किसी देश के वार्षिक वित्तीय विवरणों में पारिस्थितिक/पर्यावरणीय लागतों को शामिल नहीं करना

    (c) आधारिक संरचना विकसित करते समय अनर्थकारी पारिस्थितिक दुष्परिणामों की उपेक्षा करना

    (d) किसी सरकारी परियोजना/कार्यक्रम में पर्यावरणीय लागतों के लिए अनिवार्य उपबंध करना

    उत्तर: (A)

    • ग्रीनवॉशिंग किसी उत्पाद, सेवा, प्रौद्योगिकी या कंपनी के अभ्यास के पर्यावरणीय लाभों के बारे में एक निराधार या भ्रामक दावा करने का अभ्यास है। ग्रीनवाशिंग किसी कंपनी को अधिक पर्यावरण अनुकूल दिखने का प्रयास करती है जबकि वास्तव में वह कंपनी इतनी अधिक पर्यावरण अनुकूल नहीं होती। अतः विकल्प (a) सही है।

    https://www.drishtiias.com/current-affairs-news-analysis-editorials/news-analysis/10-12-2018

    31. निम्नलिखित कथनों पर विचार कीजिये:

    1. US पेडरल रिज़र्व की सख्त मुद्रा नीति पूँजी पलायन की ओर ले जा सकती है।
    2. पूँजी पलायन वर्तमान विदेशी वाणिज्यिक ऋणग्रहण (External Commercial Borrowings –ECBs) वाली फर्मों की ब्याज लागत को बढ़ा सकता है।
    3. घरेलू मुद्रा का अवमूल्यन, ECBs से संबद्ध मुद्रा जोखिम को घटाता है।

    उपर्युक्त कथनों में कौन-से सही हैं?

    (a) केवल 1 और 2

    (b)  केवल 2 और 3

    (c) केवल 1 और 3

    (d) 1, 2 और 3

    उत्तर:  (a)

    व्याख्या:

    • सख्त या संकुचनकारी मौद्रिक नीति एक मौद्रिक उपाय है जिसका अर्थ है केंद्रीय बैंक द्वारा मौद्रिक विस्तार की दर में कमी। यह एक सूक्ष्म आर्थिक उपकरण है जिसे केंद्रीय बैंकों या सरकारी अंतःक्षेपों द्वारा निर्मित की गई उद्गामी मुद्रास्फीति या अन्य आर्थिक विकृतियों से निपटने हेतु डिज़ाइन किया गया है। 
    • केंद्रीय बैंक छूट दर और संघीय निधि दर में नीतिगत परिवर्तनों के माध्यम से अल्पकालिक ब्याज दरों को बढ़ाकर नीति को सख्त करता है या मुद्रा को संकुचित करता है। ब्याज दरों में वृद्धि से उधार लेने की लागत बढ़ जाती है और प्रभावी रूप से इसके आकर्षण में कमी आती है।
    • केंद्रीय बैंक छूट दर और संघीय निधि दर में नीतिगत परिवर्तनों के माध्यम से अल्पकालिक ब्याज दरों को बढ़ाकर नीति को मजबूत करता है या पैसे को तंग करता है। ब्याज दरों में वृद्धि से उधार लेने की लागत बढ़ जाती है और प्रभावी रूप से इसके आकर्षण में कमी आती है।

    32. निम्नलिखित राज्यों पर विचार कीजिये:

    1. आंध्र प्रदेश
    2. केरल
    3. हिमाचल प्रदेश
    4. त्रिपुरा

    उपर्युक्त में से कितने आम तौर पर चार-उत्पादक राज्य के रूप में जाने जाते हैं?

    (a) केवल एक राज्य 

    (b) केवल दो राज्य

    (c) केवल तीन राज्य 

    (d) सभी चारों राज्य

    उत्तर: (c)

    व्याख्या:

    भारतीय चाय बोर्ड की वार्षिक रिपोर्ट वर्ष 2019-2020 के अनुसार:

    सामान्यतः चाय उत्पादक राज्य असम, त्रिपुरा, पश्चिम बंगाल, तमिलनाडु, केरल, हिमाचल प्रदेश हैं।

    2

    • अतः विकल्प (b) सही है।

    33. निम्नलिखित कथनों पर विचार कीजिये:

    1. भारत में, साख क्षमता-निर्धारण एजेंसियाँ (क्रेडिट रेटिंग एजेंसीज़) भारतीय रिज़र्व बैंक द्वारा विनियमित होती हैं।
    2. ICRA नाम से जानी जाने वाली क्षमता-निर्धारण एजेंसी एक पब्लिक लिमिटेड कंपनी है।
    3. ब्रिकवर्क रेटिंग्स एक भारतीय साख क्षमता-निर्धारण एजेंसी है।

    उपर्युक्त कथनों में कौन-से सही हैं?

    (a) केवल 1 और 2

    (b)  केवल 2 और 3

    (c) केलव 1 और 3

    (d)  1, 2 और 3

    उत्तर: (b)

    व्याख्या:

    • भारत में सभी क्रेडिट रेटिंग एजेंसियों को भारतीय प्रतिभूति और विनिमय बोर्ड अधिनियम, 1992 के सेबी (क्रेडिट रेटिंग एजेंसियांँ) विनियम, 1999 द्वारा विनियमित किया जाता है। अतः कथन 1 सही नहीं है।
    • ICRA लिमिटेड (पूर्व में इन्वेस्टमेंट इंफॉर्मेशन एंड क्रेडिट रेटिंग एजेंसी ऑफ इंडिया लिमिटेड) की स्थापना 1991 में प्रमुख वित्तीय / निवेश संस्थानों, वाणिज्यिक बैंकों और वित्तीय सेवा कंपनियों द्वारा स्वतंत्र और पेशेवर निवेश सूचना और क्रेडिट रेटिंग एजेंसी के रूप में की गई थी।
    • वर्तमान ICRA और उसकी सहायक कंपनियांँ मिलकर ICRA ग्रुप ऑफ कंपनीज़ (ग्रुप ICRA) बनाती हैं। ICRA एक पब्लिक लिमिटेड कंपनी है, जिसके शेयर बॉम्बे स्टॉक एक्सचेंज और नेशनल स्टॉक एक्सचेंज में सूचीबद्ध हैं। अतः कथन 2 सही है
    • ब्रिकवर्क रेटिंग, सेबी पंजीकृत क्रेडिट रेटिंग एजेंसी को भी आरबीआई द्वारा मान्यता प्राप्त है। अतः कथन 3 सही है।

    अतः विकल्प (b) सही है

    34. ‘बैंक बोर्ड ब्यूरो (BBB)’ के सन्दर्भ में, निम्नलिखित में कौन-से कथन सही हैं?

    1. RBI का गवर्नर BBB का चेयरमैन होता है।
    2. BBB, सार्वजनिक क्षेत्रक बैंकों के अध्यक्षों के चयन के लिए संस्तुति करता है।
    3. BBB, सार्वजनिक क्षेत्रक बैंकों को कार्यनीतियों और पूँजी-वर्धन योजनाओं को विकसित करने में मदद करता है।

    नीचे दिए गए कूट का प्रयोग कर सही उत्तर चुनिए:

    (a) केवल 1 और 2

    (b)  केवल 2 और 3

    (c) केवल 1 और 3

    (d) 1, 2 और 3

    उत्तर: (b)

    व्याख्या:

    • बैंक बोर्ड ब्यूरो (BBB) की उत्पत्ति 'भारत में बैंकों के बोर्डों के शासन की समीक्षा समिति, मई 2014 (अध्यक्ष - पी जे नायक)' की सिफारिशों में हुई है।
    • गठन: वर्ष 2016 में सरकार ने सार्वजनिक क्षेत्र के बैंकों (PSB) और राज्य के स्वामित्व वाले वित्तीय संस्थानों के पूर्णकालिक निदेशकों और गैर-कार्यकारी अध्यक्षों की नियुक्ति हेतु सिफारिश करने के लिये पेशेवरों और अधिकारियों के एक निकाय के रूप में BBB के गठन को मंज़ूरी दी।
      • यह एक स्वायत्त सलाहकारी निकाय है।
      • बैंक बोर्ड ब्यूरो में अध्यक्ष, तीन पदेन सदस्य सचिव, सार्वजनिक उद्यम विभाग, वित्तीय सेवा विभाग के सचिव और भारतीय रिज़र्व बैंक के डिप्टी गवर्नर और पांच विशेषज्ञ सदस्य शामिल हैं, जिनमें से दो निजी क्षेत्र से हैं।
      • बैंक बोर्ड ब्यूरो के अध्यक्ष की नियुक्ति मंत्रिमंडल की नियुक्ति समिति द्वारा की जाती है
    • बैंक बोर्ड ब्यूरो सार्वजनिक क्षेत्र के बैंकों को रणनीति विकसित करने और पूंजी जुटाने में मदद करता है

    अतः विकल्प (b) सही है 

    https://banksboardbureau.org.in/bureau-profile/

    35. परिवर्तनीय बॉन्ड के सन्दर्भ में, निम्नलिखित कथनों पर विचार कीजिये:

    1. चूँकि बॉन्ड को ईक्विटी के लिए बदलने का विकल्प है, परिवर्तनीय बॉन्ड अपेक्षाकृत कम ब्याज दर का भुगतान करते हैं।
    2. ईक्विटी के लिए बदलने का विकल्प बॉन्ड-धारक को बढ़ती हुई उपभोक्ता कीमतों से सहलग्नता (इंडेक्सेशन) की मात्रा प्रदान करता है।

    उपर्युक्त कथनों में कौन-सा/से सही है/हैं?

    (a) केवल 1

    (b)  केवल 2

    (c) 1 और 2 दोनों  

    (d)  न तो 1, न ही 2

    उत्तर: (c)

    व्याख्या:

    • एक परिवर्तनीय बॉण्ड एक निश्चित आय वाली कॉर्पोरेट ऋण सुरक्षा है जो ब्याज भुगतान प्राप्त करती है, लेकिन इसे सामान्य स्टॉक या इक्विटी शेयरों की पूर्व निर्धारित संख्या में परिवर्तित किया जा सकता है।
    • कंपनियांँ ऋण पर कूपन दर (कम ब्याज दर) तथा मन्दन को कम करने के लिये परिवर्तनीय बॉण्ड ज़ारी करती हैं। अतः कथन 1 सही है
    • वे निवेशकों के लिये अधिक आकर्षक हो सकते हैं क्योंकि परिवर्तनीय बॉण्ड स्टॉक की कीमत को भविष्य में पूंजी अभिमुल्यन के माध्यम से विकास क्षमता प्रदान करते हैं। इसलिये उपभोक्ता कीमतों में बढ़ोतरी से निवेशकों को फायदा होगा। अत: कथन 2 सही है।
    • परिवर्तनीय बॉण्ड आमतौर पर सीधे कॉरपोरेट बॉण्ड की तुलना में कम ब्याज दरों का भुगतान करते हैं- ब्याज व्यय में बचत महत्त्वपूर्ण हो सकती है।
    • निवेशक कम ब्याज भुगतान स्वीकार करते हैं क्योंकि परिवर्तन विकल्प स्टॉक मूल्य में वृद्धि से लाभ उठाने का अवसर प्रदान करता है।

    https://www.forbes.com/advisor/investing/convertible-bonds/

    36. निम्नलिखित पर विचार कीजिये:

    1. एशियाई अवसंरचना निवेश बैंक (एशियन इन्फ्रास्ट्रक्चर इन्वेस्टमेंट बैंक)
    2. प्रक्षेपास्त्र प्रौद्योगिकी नियंत्रण व्यवस्था (मिसाइल टेक्नोलॉजी कन्ट्रोल रिजीम)
    3. शंघाई सहयोग संगठन (शंघाई कोऑपरेशन ऑर्गेनाईज़ेशन)

    भारत उपर्युक्त में से किसका/किनका सदस्य है?

    (a) केवल 1 और 2

    (b)  केवल 3

    (c) केवल 2 और 3

    (d)  1, 2 और 3

    उत्तर: (d)

    व्याख्या:

    • मिसाइल प्रौद्योगिकी नियंत्रण व्यवस्था (MTCR) मिसाइल और मानव रहित हवाई वाहन प्रौद्योगिकी (जो 300 किमी से अधिक दूरी  के लिये 500 किलोग्राम से अधिक पेलोड ले जाने में सक्षम) के प्रसार को रोकने के लिये 35 देशों के बीच एक अनौपचारिक और स्वैच्छिक साझेदारी है, है।
      • भारत को वर्ष 2016 में 35वें सदस्य के रूप में मिसाइल प्रौद्योगिकी नियंत्रण व्यवस्था में शामिल किया गया था।

    https://www.drishtiias.com/to-the-points/Paper2/multilateral-export-control-regimes

    • AIIB एक बहुपक्षीय विकास बैंक है जिसका उद्देश्य एशिया में सामाजिक और आर्थिक परिणामों में सुधार करना है। AIIB की सदस्यता विश्व बैंक या एशियाई विकास बैंक के सभी सदस्यों के लिये खुली है, इसे क्षेत्रीय और गैर-क्षेत्रीय सदस्यों में विभाजित किया गया है।
      • भारत इसका दूसरा सबसे बड़ा शेयरधारक है, जिसने 8.4 बिलियन अमेरिकी डॉलर का योगदान दिया है।

    https://www.drishtiias.com/important-institutions/drishti-specials-important-institutions-international-institution/asian-infrastructure-investment-bank 

    • SCO एक स्थायी अंतर-सरकारी अंतर्राष्ट्रीय संगठन है। यह एक यूरेशियाई राजनीतिक, आर्थिक और सैन्य संगठन है, जिसका लक्ष्य इस क्षेत्र में शांति, सुरक्षा तथा स्थिरता को बनाए रखना है।
      • भारत और पाकिस्तान 9 जून, 2017 को पूर्ण सदस्य के रूप में SCO में शामिल हुए।

    https://www.drishtiias.com/important-institutions/drishti-specials-important-institutions-international-institution/important-international-institutions-shanghai-cooperation-organisation-sco 

    अतः विकल्प (d) सही है।

    37. निम्नलिखित कथनों पर विचार कीजिये:

    1. हाल के वर्षों में वियतनाम विश्व में सबसे तेज़ी से बढ़ती हुई अर्थव्यवस्थाओं में से एक रहा है।
    2. वियतनाम का नेतृत्व बहु-दलीय राजनीतिक प्रणाली के द्वारा होता है।
    3. वियतनाम का आर्थिक विकास विश्वव्यापी पूर्ति शृंखलाओं के साथ इसके एकीकरण और निर्यात पर मुख्य ध्यान होने से जुड़ा है।
    4. लंबे समय से वियतनाम की निम्न श्रम लागतों और स्थिर विनिमय दरों ने वैश्विक निर्माताओं को आकर्षित किया है।
    5. हिंद-प्रशांत क्षेत्र का सर्वाधिक उत्पादक म-सेवा सेक्टर वियतनाम में है।

    उपर्युक्त कथनों में कौन-से सही हैं?

    (a) 2 और 4

    (b)  3 और 5

    (c) 1, 3 और 4

    (d)  1 और 2

    उत्तर: (c)

    व्याख्या:

    • वियतनाम एक-दलीय प्रणाली वाला देश है, जहाँ वियतनाम की सत्तारूढ़ कम्युनिस्ट पार्टी (CPV) दशकों से हावी है। अतः कथन 2 सही नहीं है।
    • 'https://statisticstimes.com' के अनुसार, वर्ष 2020 और 2021 में वियतनाम की विकास दर रैंक क्रमशः 15 और 38 है। अत: कथन 1 सही है।
    • विनिर्माण उद्योग कई प्रमुख कारकों द्वारा संचालित होता है। सबसे पहले, वियतनाम को प्रतिस्पर्द्धी श्रम लागत के साथ कम लागत वाला निर्माता माना जाता है। औसतन, वियतनाम की श्रम लागत चीन की श्रम लागत से आधी है।
    • वर्ष 2010 के बाद से, वियतनाम की मुद्रा की सराहना की गई है और वर्ष 2015 से, सरकार ने डॉलर के मुकाबले वियतनामी डोंग (VND) को वास्तविक रूप से स्थिर रखा है। अत: कथन 4 सही है।
    • अंतर्राष्ट्रीय मुद्रा कोष (IMF) के अनुसार, "COVID-19 के बावजूद, वियतनाम की अर्थव्यवस्था लचीली बनी हुई है"। वर्ष 2020 में, इसमें 2.9% और वर्ष 2021 का विस्तार हुआ। यह एशिया की सबसे तेजी से बढ़ती अर्थव्यवस्थाओं में से एक है; यह अमेरिका के साथ चल रहे व्यापार युद्ध में चीन द्वारा खोए गए अधिकांश व्यवसाय को लेने में कामयाब रहा है, जबकि अभी भी दोनों देशों के साथ ठोस संबंध स्थापित कर रहा है। वियतनाम की अर्थव्यवस्था बढ़ती रहेगी और देश वैश्विक आपूर्ति श्रृंखला में महत्त्वपूर्ण भूमिका निभाएगा। अत: कथन 3 सही है।

    38. भारत में, निम्नलिखित में कौन मुद्रास्फीति को नियंत्रित कर कीमत स्थिरता बनाए रखने के लिए उत्तरदायी है?

    (a) उपभोक्ता मामले विभाग

    (b) व्यय प्रबंधन आयोग

    (c) वित्तीय स्थिरता और विकास परिषद्

    (d) भारतीय रिज़र्व बैंक

    उत्तर: (D)

    व्याख्या:

    मौद्रिक नीति का प्राथमिक उद्देश्य विकास के उद्देश्य को ध्यान में रखते हुए मूल्य स्थिरता बनाए रखना है। सतत् विकास के लिये मूल्य स्थिरता एक आवश्यक पूर्व शर्त है।

    भारतीय रिज़र्व बैंक (RBI) में  मौद्रिक नीति के संचालन की ज़िम्मेदारी निहित है। यह ज़िम्मेदारी भारतीय रिज़र्व बैंक अधिनियम, 1934 के तहत अनिवार्य है।

    मई 2016 में, भारतीय रिज़र्व बैंक (RBI) अधिनियम, 1934 को लचीला मुद्रास्फीति लक्ष्यीकरण ढांँचे (Flexible Inflation Targeting Framework) के कार्यान्वयन के लिये एक वैधानिक आधार प्रदान करने हेतु संशोधित किया गया था।

    अतः विकल्प (d) सही है।

    https://www.drishtiias.com/daily-updates/daily-news-analysis/monetary-policy-committee-rbi 

    https://www.rbi.org.in/scripts/FS_Overview.aspx?fn=2752

    39. नॉन-पंजिबल टोकेंस (Non-Fungible Tokens–NFTs) के सन्दर्भ में, निम्नलिखित कथनों पर विचार कीजिये:

    1. वे भौतिक परिसंपत्तियों के अंकीय निरूपण (डिजिटल रिप्रेज़ेंटेशन) को सुकर बनाते हैं।
    2. वे अनन्य क्रिप्टोग्राफिक टोकेंस हैं जो किसी ब्लॉकचैन में विद्यमान हैं।
    3. उनका, तुल्यता पर, व्यापार या विनिमय किया जा सकता है और इसलिए उनका वाणिज्यिक लेन-देन के माध्यम के रूप में इस्तेमाल किया जा सकता है।

    उपर्युक्त कथनों में कौन-से सही हैं?

    (a) केवल 1 और 2

    (b)  केवल 2 और 3

    (c) केवल 1 और 3

    (d)  1, 2 और 3

    उत्तर: (A)

    व्याख्या:

    • कोई भी चीज जिसे डिजिटल रूप में बदला जा सकता है वह NFTs हो सकती हैं।
    • ड्रॉइंग, फोटो, वीडियो, GIF, संगीत, इन-गेम आइटम, सेल्फी और यहाँँ तक कि एक ट्वीट, सब कुछ एक NFT में बदला जा सकता है, जिसे बाद में क्रिप्टोकरेंसी का उपयोग करके ऑनलाइन कारोबार किया जा सकता है।
    • अगर कोई  व्यक्ति अपनी डिजिटल संपत्ति को NFTs में परिवर्तित करता है, तो उसे ब्लॉकचैन द्वारा संचालित स्वामित्व का प्रमाण मिलेगा।

    अत: कथन 1 और 2 सही हैं।

    • NFTs नॉन-फंजिबल टोकेंस हैं, जिसका अर्थ है कि एक NFT का मूल्य दूसरे के बराबर नहीं है।
    • नॉन-फंजिबल का अर्थ है कि  NFT परस्पर विनिमेय नहीं हैं।  प्रत्येक NFT की UNIQUE पहचान होती है, जो इसे नॉन-फंजिबल और अद्वितीय बनाती है।

    अत: कथन 3 सही नहीं है।

    https://www.drishtiias.com/daily-updates/daily-news-analysis/non-fungible-tokens

    40. निम्नलिखित युग्मों पर विचार कीजिये:

    जलाशय राज्य
    1. घाटप्रभा तेलंगाना
    2. गांधी सागर मध्य प्रदेश
    3. इंदिरा सागर  आंध्र प्रदेश
    4. मैथोन छत्तीसगढ़

    उपर्युक्त में से कितने युग्म सही सुमेलित नहीं हैं?

    (a) केवल एक युग्म 

    (b)  केवल दो युग्म

    (c) केवल तीन युग्म 

    (d)  सभी चारों युग्म

    उत्तर: (c)

    व्याख्या:

    घाटप्रभा में जलविद्युत और सिंचाई बांँध हिडकल है।

    • हिडकल बांँध कर्नाटक के बेलगावी ज़िले में स्थित है। यह बांँध वर्ष 1977 में बनकर तैयार हुआ था। इसे बहुउद्देशीय परियोजना बनाने के लिये बांँध पर एक जलाशय भी बनाया गया था।
    • चंबल नदी (मध्य प्रदेश) पर गांधी सागर बांँध राष्ट्रीय महत्त्व के पांँच जलाशयों में से एक है।
    • इंदिरा सागर (पोलावरम) परियोजना आंध्र प्रदेश में पश्चिम गोदावरी ज़िले के पोलावरम मंडल के रामय्यापेट गाँव के निकट गोदावरी नदी पर स्थित है।
      • हालाँकि, प्रश्न में दिया गया इंदिरा सागर, मध्य प्रदेश में है क्योंकि आंध्र प्रदेश के इंदिरा सागर में पोलावरम शामिल होगा।
    • मैथन बांँध धनबाद (झारखंड) के कोयला शहर से लगभग 48 किलोमीटर की दूरी पर स्थित है।

    सही युग्म

    जलाशय राज्य
    1. घाटप्रभा कर्नाटक
    2. गांधी सागर मध्य प्रदेश
    3. इंदिरा सागर मध्य प्रदेश
    4. मैथन झारखंड

    अतः विकल्प (c) सही है।

    https://www.drishtiias.com/daily-updates/daily-news-analysis/ghataprabha-river 

    https://www.drishtiias.com/daily-updates/daily-news-analysis/state-of-india-s-dams 

    41. भारत सरकार अधिनियम, 1919 में, प्रांतीय सरकार के कार्य ‘‘आरक्षित (रिज़र्व्ड)’’ और ‘‘अंतरित (ट्रांसफर्ड)’’ विषयों के अंतर्गत बाँटे गए थे। निम्नलिखित में कौन-से ‘‘आरक्षित’’ विषय माने गए थे?

    1. न्याय प्रशासन
    2. स्थानीय स्वशासन
    3. भू-राजस्व
    4. पुलिस

    नीचे दिए कूट का प्रयोग कर सही उत्तर चुनिए:

    (a) 1, 2 और 3

    (b)  2, 3 और 4

    (c) 1, 3 और 4

    (d) 1, 2 और 4

    उत्तर: (c)

    व्याख्या:

    भारत सरकार अधिनियम, 1919 के अनुसार विषयों को दो सूचियों में विभाजित किया गया था: "आरक्षित सूची" जिसमें कानून और व्यवस्था, वित्त, भूमि राजस्व, सिंचाई, आदि जैसे विषय शामिल थे, जबकि शिक्षा, स्वास्थ्य, स्थानीय सरकार, उद्योग, कृषि, उत्पाद शुल्क, आदि जैसे विषय  "स्थानांतरित सूची" में शामिल थे।

    42. मध्यकालीन भारत में, शब्द ‘‘फणम’’ किसे निर्दिष्ट करता था?

    (a) पहनावा 

    (b) सिक्के

    (c) आभूषण

    (d) हथियार

    उत्तर:(b)

    व्याख्या:

    मध्यकालीन त्रावणकोर में फैनम और चकराम सिक्के मुद्रा की नियमित इकाई थे और व्यापार के लिये बड़े पैमाने पर उपयोग किये जाते थे।

    43. निम्नलिखित स्वतंत्रता सेनानियों पर विचार कीजिये:

    1. बारींद्र कुमार घोष
    2. जोगेश चंद्र चटर्जी
    3. रास बिहारी बोस

    उपर्युक्त में से कौन गदर पार्टी के साथ सक्रिय रूप से जुड़ा था/जुड़े थे?

    (a) 1 और 2

    (b)  केवल 2

    (c) 1 और 3

    (d)  केवल 3

    उत्तर: (d)

    व्याख्या

    • बरिंद्र कुमार घोष (बरिंद्र घोष) बंगाली क्रांतिकारी आंदोलन, जुगंतर/युगांतर के संस्थापक सदस्य थे। वह गदर पार्टी से नहीं जुड़े थे। 
    • जोगेश चंद्र अनुशीलन समिति के सदस्य बने। वह हिंदुस्तान रिपब्लिकन एसोसिएशन (HRA) (1924 में) के संस्थापक सदस्यों में से एक थे। वह गदर पार्टी से नहीं जुड़े थे। 
    • रास बिहारी बोस ब्रिटिश राज के खिलाफ भारतीय क्रांतिकारी नेता थे। वह गदर विद्रोह के प्रमुख आयोजकों में से एक थे।

    अतः विकल्प (d) सही है।

    44. क्रिप्स मिशन के प्रस्तावों के सन्दर्भ में, निम्नलिखित कथनों पर विचार कीजिये:

    1. संविधान सभा में प्रांतीय विधान सभाओं और साथ ही भारतीय रियासतों द्वारा नामित सदस्य होंगे।
    2. नया संविधान स्वीकार करने के लिए जो भी प्रांत तैयार नहीं होगा, उसे यह अधिकार होगा कि अपनी भावी स्थिति के बारे में ब्रिटेन के साथ अलग संधि पर हस्ताक्षर करे।

    उपर्युक्त कथनों में कौन-सा/से सही है/हैं?

    (a) केवल 1

    (b)  केवल 2

    (c) 1 और 2 दोनों 

    (d)  न तो 1, न ही 2

    उत्तर: (b)

    व्याख्या:

    • क्रिप्स मिशन के अनुसार, देश का एक नया संविधान बनाने के लिये एक संविधान सभा का गठन किया जाएगा। इस सभा में प्रांतीय विधानसभाओं द्वारा चुने गए सदस्य और राजाओ द्वारा मनोनीत सदस्य भी होंगे। अतः कथन 1 सही नहीं है। 
    • इसके अलावा मिशन ने प्रस्तावित किया कि कोई भी प्रांत भारतीय प्रभुत्व में शामिल होने के इच्छुक नहीं है तो एक अलग संघ बना सकता है जिसका एक अलग संविधान हो सकता है। अतः कथन 2 सही है।

    https://www.drishtiias.com/hindi/mains-practice-question/question-164 

    45. भारतीय इतिहास के सन्दर्भ में, निम्नलिखित मूलग्रंथों पर विचार कीजिये:

    1. नेत्तिपकरण
    2. परिशिष्टपर्वन
    3. अवदानशतक
    4. त्रिशष्टिलक्षण महापुराण

    उपर्युक्त में कौन-से जैन ग्रंथ हैं?

    (a) 1, 2 और 3

    (b)  केवल 2 और 4

    (c) 1, 3 और 4

    (d)  2, 3 और 4

    उत्तर: (b)

    व्याख्या:

    • नेट्टिपकारा एक पौराणिक बौद्ध ग्रंथ है, जिसे कभी-कभी थेरवाद बौद्ध धर्म के पाली कैनन के खुदाका निकाय में शामिल किया गया था। 
    • परिशिष्टपर्वन हेमचन्द्र द्वारा रचित 12वीं शताब्दी का संस्कृत महाकाव्य है जो प्रारंभिक जैन शिक्षकों के इतिहास का विवरण प्रस्तुत करता है। 
    • अवदानशतक एक सौ बौद्ध किंवदंतियों का संस्कृत में एक संकलन है, जो लगभग एक ही समय से संबंधित है। त्रिशष्ठिलक्षण महापुराण एक प्रमुख जैन ग्रंथ है जिसकी रचना मुख्य रूप से आचार्य जिनसेना ने राष्ट्रकूट के शासन के दौरान की थी। 

    अतः विकल्प (b) सही है।

    46. भारतीय इतिहास के सन्दर्भ में, निम्नलिखित युग्मों पर विचार कीजिये:

    ऐतिहासिक व्यक्ति किस रूप में जाने गए
    1. आर्यदेव जैन विद्वान
    2. दिग्नाग बौद्ध विद्वान
    3. नाथमुनि वैष्णव विद्वान

    उपर्युक्त युग्मों में से कितने युग्म सही सुमेलित हैं?

    (a) कोई भी युग्म नहीं 

    (b)  केवल एक युग्म

    (c) केवल दो युग्म 

    (d)  सभी तीन युग्म

    उत्तर: (c)

    व्याख्या:

    • आर्यदेव महायान बौद्ध भिक्षु, नागार्जुन के शिष्य और मध्यमा दार्शनिक थे। 
    • दिग्नागा भारतीय बौद्ध विद्वान और भारतीय तर्कशास्त्र के बौद्ध संस्थापकों में से एक थे। 
    • श्री रंगनाथमुनि जिन्हें लोकप्रिय रूप से श्रीमन नाथमुनि (823 सीई-951 सीई) के नाम से जाना जाता है, एक वैष्णव धर्मशास्त्री थे जिन्होंने नलयिर दिव्य प्रबंधम को एकत्र और संकलित किया था। अत: केवल युग्म 2 और 3 सही सुमेलित हैं। पहला युग्म गलत सुमेलित है।

    47. भारतीय इतिहास के सन्दर्भ में, निम्नलिखित कथनों पर विचार कीजिये:

    1. भारत पर पहला मंगोल आक्रमण जलालुद्दीन खिलज़ी के राज्यकाल में हुआ।
    2. अलाउद्दीन खिलज़ी के राज्य-काल में, एक मंगोल आक्रमण दिल्ली तक आ पहुँचा और उस शहर पर घेरा डाल दिया।
    3. मुहम्मद-बिन-तुगलक मंगोलों से अपने राज्य के कुछ उत्तरी-पश्चिमी भाग अस्थायी रूप से हार गया था।

    उपर्युक्त कथनों में कौन-सा/से सही है/हैं

    (a) 1 और 2

    (b)  केवल 2

    (c) 1 और 3

    (d) केवल 3

    उत्तर: (b)

    व्याख्या:

    • 1240-66 के बीच मंगोलों ने पहली बार भारत के विलय की नीति अपनाई और इसके साथ ही  दिल्ली के साथ आपसी 'गैर-आक्रामकता समझौते का स्वर्णिम चरण समाप्त हो गया। 1241 में तायर बहादुर के अधीन मंगोलों ने लाहौर पर आक्रमण किया और शहर को पूरी तरह से नष्ट कर दिया। अतः कथन 1 गलत है।
    • 1299 में मंगोलों ने मंगोल शासक दवा खान के पुत्र कुतलुग खान (कुतलुग ख्वाजा) के नेतृत्व में भारत पर आक्रमण किया। मंगोलों द्वारा दिल्ली को तबाह करने का यह पहला प्रयास था। उनके निकट आने की बात सुनकर अलाउद्दीन ने अफरा तफरी में एक सेना तैयार की और सिरी के बाहर इसे तैनात किया। मंगोलों ने दिल्ली से छह मील उत्तर में स्थित किल्ली नामक स्थान पर प्रवेश किया। अतः कथन 2 सही है।
    • अंतिम महत्त्वपूर्ण मंगोल आक्रमण सुल्तान मुहम्मद तुगलक के शासनकाल के दौरान तरमाशिरिन के नेतृत्व में हुआ था। गयासुद्दीन तुगलक ने तरमाशिरिन के खिलाफ आक्रमण किया और उसे  सिंधु के पार वापस भेज दिया जो मंगोलों की सीमा बनी हुई थी। अतः कथन 3 गलत है।

    48. भारतीय इतिहास के सन्दर्भ में, निम्नलिखित में से कौन ‘‘कुलाह-दारन’’ कहलाते थे?

    (a) अरब व्यापारी 

    (b)  कलंदर

    (c) फारसी खुशनवीस 

    (d)  सबयद

    उत्तर: (d)

    व्याख्या:

    • सैय्यदों ने उनकी पुत्री फातिमा के माध्यम से पैगंबर का वंशज़ होने का दावा किया। मुस्लिम समाज में उनका विशेष सम्मान था। 
    • तैमूर ने भारत पर आक्रमण के दौरान सैय्यदों के जीवन की रक्षा की, हालांँकि उनकी नीति सामान्य वध की थी। 
    • राज्य के राजस्व के दुरुपयोग के आरोपी सैय्यद को सिकंदर लोदी ने रिहा कर दिया और उसे अपने बेईमानी से प्राप्त  लाभ को अपने पास रखने की अनुमति दी गई। 
    • सैय्यद एक नुकीली टोपी (कुलह) लगाते थे इसलिये उन्हें कुलह-दारन के नाम से जाना जाता था।

    49. भारतीय इतिहास के सन्दर्भ में, निम्नलिखित कथनों पर विचार कीजिये:

    1. डच लोगों ने पूर्वी क्षेत्रों में गजपति शासकों द्वारा प्रदान की गई जमीनों पर अपनी पैक्टरियाँ/गोदाम स्थापित किए।
    2. अल्फोंसो दे अलबुकर्क ने बीजापुर सल्तनत से गोआ को छीन लिया था।
    3. अंग्रेज़ी ईस्ट इंडिया कंपनी ने मद्रास में विजयनगर साम्राज्य के एक प्रतिनिधि से पट्टे पर ली गई जमीन के एक प्लॉट पर पैक्टरी स्थापित की थी।

    उपर्युक्त कथनों में कौन-से सही हैं?

    (a) केवल 1 और 2

    (b)  केवल 2 और 3

    (c) केवल 1 और 3

    (d)  1, 2 और 3

    उत्तर: (b)

    व्याख्या:

    • ओड़िशा में गंग राजवंश के बाद एक और गौरवशाली राजवंश आया, जिसे सूर्यवंशी गजपति कहा जाता है। इस वंश के अंतिम शासक कखरुआ देव की 1541 में गोविंद विद्याधर ने हत्या कर दी थी, जिन्होंने भोई वंश की स्थापना की थी। अतः कथन 1 सही नहीं है।
    • अल्फोंसो डी अल्बुकर्क के पास 23 युद्धपोत और लगभग 1000 सैनिक थे। जनवरी 1510 में उसने गोवा पर आक्रमण किया। उस समय गोवा की सत्ता बीजापुर के शासक के हाथ में थी, जो अपने ही राज्य में विद्रोह का दमन करने में लगा हुआ था। मौके का फायदा उठाकर अल्फोंसो डी अल्बुकर्क ने गोवा पर कब्जा कर लिया। अतः कथन 2 सही है।
    • 1611 ई. में, अंग्रेज़ों ने दक्षिण भारत में मछलीपट्टनम में अपना पहला कारखाना स्थापित किया, लेकिन जल्द ही गतिविधियों का मुख्य केंद्र मद्रास में स्थानांतरित हो गया। फ्रांसिस डे ने 1639 में विजयनगर साम्राज्य के प्रतिनिधि चंद्रगिरी से मद्रास को पट्टे पर दिया था। उन्होंने वहाँ एक किलेबंद कोठी का निर्माण किया, जिसका नाम 'फोर्ट सेंट जॉर्ज' रखा गया। अतः कथन 3 सही है।

    50. कौटिल्य अर्थशास्त्र के अनुसार, निम्नलिखित में कौन-से सही हैं?

    1. न्यायिक दंड के परिणामस्वरूप कोई व्यक्ति दास हो सकता था।
    2. स्त्री दास अपने मालिक के संसर्ग से पुत्र जनन पर कानूनी तौर पर मुक्त हो जाती थी।
    3. यदि स्त्री दास का मालिक उस स्त्री से पैदा हुए पुत्र का पिता हो, तो उस पुत्र को मालिक का पुत्र होने का कानूनी हक मिलता था।

    उपर्युक्त कथनों में कौन-से सही हैं?

    (a) केवल 1 और 2

    (b)  केवल 2 और 3

    (c) केवल 1 और 3

    (d)  1, 2 और 3

    उत्तर: (d) 

    व्याख्या:

    • 'अर्थशास्त्र' में कहा गया है कि एक आदमी जन्म से, स्वेच्छा से खुद को बेचकर, युद्ध में कैद होकर, या न्यायिक दंड के परिणामस्वरूप गुलाम/दास हो सकता है। यदि एक दासी ने अपने स्वामी के बच्चे को जन्म दिया, तो वह बंधन से मुक्ति की हकदार होती थी; साथ ही, ऐसी गर्भवती दासी को गर्भावस्था की अवधि के दौरान उसकी देखभाल के लिये उचित व्यवस्था किये बिना बेचा या गिरवी नहीं रखा जा सकता था। अतः कथन 1 और 2 सही हैं।
    • जब किसी दासी द्वारा से अपने स्वामी के बच्चे को जन्म दिया जाता है, तो बच्चा और उसकी माँ दोनों को एक ही बार में स्वतंत्र माना जाएगा। यदि जीविका के लिये माँ को बंधन में रहना है, तो उसके भाई और बहन को मुक्त कर दिया जाएगा। एक बार मुक्त होने के बाद किसी पुरुष या महिला गुलाम के जीवन को बेचने या गिरवी रखने पर 12 पनास के ज़ुर्माने से दंडित किया जाएगा, सिवाय उन लोगों के जो खुद को गुलाम बनाते हैं। अतः कथन 3 सही है।

    51. ‘‘जलवायु कार्रवाई ट्रैकर (क्लाइमेट ऐक्शन ट्रैकर)’’ जो विभिन्न देशों के उत्सर्जन अपचयन के लिए दिए गए वचनों की निगरानी करता है, क्या है?

    (a) अनुसंधान संगठनों के गठबंधन द्वारा निर्मित डेटाबेस

    (b) ‘‘जलवायु परिवर्तन के अंतर्राष्ट्रीय पैनल’’ का स्कंध (विंग)

    (c) ‘‘जलवायु परिवर्तन पर संयुक्त राष्ट्र ढाँचा अभिसमय’’ के अधीन समिति

    (d) संयुक्त राष्ट्र पर्यावरण कार्यक्रम और विश्व बैंक द्वारा संवर्धित और वित्तपोषित एजेंसी

    उत्तर: (A) 

    व्याख्या: 

    क्लाइमेट एक्शन ट्रैकर (Climate Action Tracker-CAT) एक स्वतंत्र वैज्ञानिक विश्लेषण है जो सरकारी जलवायु कार्रवाई को ट्रैक करता है और इसे पेरिस समझौते के खिलाफ मापता है। यह जलवायु परिवर्तन शमन लक्ष्यों, नीतियों और कार्यों की मात्रा निर्धारित करता है तथा उनका मूल्यांकन करता है। यह MAGICC जलवायु मॉडल का उपयोग करते हुए 21वीं सदी के दौरान संभावित तापमान वृद्धि का निर्धारण करते हुए, वैश्विक स्तर पर देश की कार्रवाई को भी एकत्रित करता है। यह दो संगठनों, क्लाइमेट एनालिटिक्स और न्यू क्लाइमेट इंस्टीट्यूट के सहयोग से नीति निर्माताओं को स्वतंत्र विश्लेषण प्रदान करता है। अतः विकल्प (a) सही है।

    स्रोत: CAT Website

    52. निम्नलिखित कथनों पर विचार कीजिये:

    1. ‘‘जलवायु समूह (दि क्लाइमेट ग्रुप)’’ एक अंतर्राष्ट्रीय ग़ैर-लाभकारी संगठन है जो बड़े नेटवर्क बना कर जलवायु क्रिया को प्रेरित करता है और उन्हें चलाता है।
    2. अंतर्राष्ट्रीय ऊर्जा एजेंसी ने जलवायु समूह की भागीदारी में एक वैश्विक पहल "EP100" प्रारंभ की।
    3. EP100, ऊर्जा दक्षता में नवप्रवर्तन को प्रेरित करने एवं उत्सर्जन न्यूनीकरण लक्ष्यों को प्राप्त करते हुए प्रतिस्पर्धात्मकता बढ़ाने के लिए प्रतिबद्ध अग्रणी कंपनियों को साथ लाता है।
    4. कुछ भारतीय कंपनियाँ EP100 की सदस्य हैं।
    5. अंतर्राष्ट्रीय ऊर्जा एजेंसी ‘‘अंडर 2 कोएलिशन’’ का सचिवालय है।

    उपर्युक्त कथनों में कौन-से सही हैं?

    (a) 1, 2, 4 और 5

    (b)  केवल 1, 3 और 4

    (c) केवल 2, 3 और 5  

    (d)  1, 2, 3, 4 और 5

    उत्तर: (B) 

    व्याख्या: 

    • जलवायु समूह (The Climate Group) 2003 में स्थापित एक अंतरार्ष्ट्रीय गैर-लाभकारी संगठन है, जो बड़े नेटवर्क को शक्ति प्रदान करते हैं, जलवायु कार्रवाई को प्रेरित करता है और उन्हें तीव्रता से चलाता है। 
    • “EP100” जलवायु समूह के नेतृत्व में शुरू की गई एक वैश्विक पहल है, जो ऊर्जा दक्षता सुधारों को मापने और रिपोर्ट करने के लिये प्रतिबद्ध 120 से अधिक ऊर्जा स्मार्ट व्यवसायों को एक साथ लाती है। 
    • EP100 कंपनियाँ  नवाचार को प्रेरित करने एवं उत्सर्जन न्यूनीकरण लक्ष्यों को पूरा करते हुए अन्य कंपनियों को प्रेरणा और सहायता भी प्रदान करती हैं। 
    • कुछ भारतीय कंपनियां जैसे - स्वराज एंजिंस लिमिटेड, महिंद्रा एंड महिंद्रा आदि EP100 की सदस्य हैं। 
    • ‘Under 2 Coalition’ पेरिस समझौते के अनुरूप उत्सर्जन को कम करने के लिये प्रतिबद्ध राज्य और क्षेत्रीय सरकारों का सबसे बड़ा वैश्विक नेटवर्क है। जलवायु समूह (The Climate Group) Under 2 Coalition का सचिवालय है।

    अतः विकल्प (b) सही है।

    स्रोत: The Climate Group Website

    53. ‘‘यदि वर्षावन और उष्णकटिबंधीय वन पृथ्वी के फेफड़े हैं, तो निश्चित ही आर्द्रभूमियाँ इसके गुर्दों की तरह काम करती हैं।’’ निम्नलिखित में से आर्द्रभूमियों का कौन-सा एक कार्य उपर्युक्त कथन को सर्वोत्तम रूप से प्रतिबिंबित करता है?

    (a) आर्द्रभूमियों के जल चक्र में सतही अपवाह, अवमृदा अंत:स्रवण और वाष्पन शामिल होते हैं।

    (b) शैवालों से वह पोषक आधार बनता है, जिस पर मत्स्य, परुषकवची (क्रश्टेशिआई), मृदुकवची (मोलस्क), पक्षी, सरीसृप और स्तनधारी फलते-फूलते हैं।

    (c) आर्द्रभूमियाँ अवसाद संतुलन और मृदा स्थिरीकरण बनाए रखने में महत्त्वपूर्ण भूमिका निभाती हैं।

    (d) जलीय पादप भारी धातुओं और पोषकों के आधिक्य को अवशोषित कर लेते हैं।

    उत्तर: (c)

    व्याख्या: 

    • आर्द्रभूमि महत्त्वपूर्ण निस्यंदक (Filter) हैं। ये अवसाद को ट्रैप करती हैं, प्रदूषकों को हटाती हैं और जल को शुद्ध करती हैं। आर्द्रभूमियाँ अपरदन को भी नियंत्रित करती हैं। इस प्रकार, आर्द्रभूमि अवसाद संतुलन और मृदा स्थिरीकरण को बनाए रखने में महत्त्वपूर्ण भूमिका निभाती हैं।

    अतः विकल्प (c) सही है।

    स्रोत : डाउन टू अर्थ 

    54. WHO के वायु गुणवत्ता दिशानिर्देशों के सन्दर्भ में, निम्नलिखित कथनों पर विचार कीजिये:

    1. PM2.5 का 24 घंटा माध्य 15µg/m3 से अधिक नहीं बढ़ना चाहिए और PM2.5 का वार्षिक माध्य 5µg/m3  से अधिक नहंी बढ़ना चाहिए।
    2. किसी वर्ष में, ओज़ोन प्रदूषण के उच्चतम स्तर प्रतिकूल मौसम के दौरान होते हैं।
    3. PM10 फेफड़े के अवरोध का वेधन कर रक्त-प्रवाह में प्रवेश कर सकता है।
    4. वायु में अत्यधिक ओज़ोन दमा को उत्पन्न कर सकती है।

    उपर्युक्त कथनों में कौन-से सही हैं?

    (a) 1, 3 और 4

    (b)  केवल 1 और 4

    (c) 2, 3 और 4

    (d)  केवल 1 और 2

    उत्तर (B) 

    व्याख्या:  

    • WHO वायु गुणवत्ता दिशानिर्देशों के तहत PM2.5 का 24 माध्य 15 माइक्रोग्राम प्रति घन मीटर से अधिक नहीं होना चाहिये और पीएम2.5 का वार्षिक औसत 5 μg/m3 से अधिक नहीं होना चाहिये। 
    • PM10 फेफड़ों में प्रवेश कर सकते हैं, जबकि PM2.5 फेफड़ों के अवरोध का वेधन कर रक्त-प्रणाली में प्रवेश कर सकता है। 
    • ज़मीनी स्तर पर ओज़ोन प्रकाश-रासायनिक धूम्र/फोटोकैमिकल स्मॉग के प्रमुख घटकों में से एक है। धूप के मौसम में ओज़ोन प्रदूषण का स्तर उच्चतम होता है। 
    • वायु में अत्यधिक ओज़ोन मानव स्वास्थ्य पर एक उल्लेखनीय प्रभाव डाल सकता है। यह सांँस लेने में समस्या पैदा कर सकता है, अस्थमा तथा फेफड़ों की बीमारियों का कारण बन सकता है।

    अतः विकल्प (b) सही है।

    स्रोत: WHO 

    55. कभी-कभी समाचारों में उल्लिखित ‘गुच्छी’ के सन्दर्भ में, निम्नलिखित कथनों पर विचार कीजिये:

    1. यह एक कवक है।
    2. यह कुछ हिमालयी वन क्षेत्रों में उगती है।
    3. उत्तर-पूर्वी भारत के हिमालय की तलहटी में इसकी वाणिज्यिक रूप से खेती की जाती है।

    उपर्युक्त कथनों में कौन-सा/से सही है/हैं?

    (a) केवल 1

    (b) केवल 3

    (c) 1 और 2

    (d) 2 और 3

    उत्तर (C) 

    व्याख्या: 

    • गुच्छी (Gucchi) मशरूम एस्कोमाइकोटा के परिवार मोरचेलासी में कवक की एक प्रजाति है। ये हल्के पीले रंग के  होते हैं। 
    • इसकी खेती व्यावसायिक रूप से  नहीं की जा सकती है और ये हिमाचल प्रदेश, उत्तरांचल तथा जम्मू-कश्मीर की तलहटी में उगते हैं।

    स्रोत : द इंडियन एक्सप्रेस 

    56. पॉलीएथिलीन टेरेफ्थलेट के सन्दर्भ में, जिसका हमारे दैनिक जीवन में बहुत व्यापक उपयोग है, निम्नलिखित कथनों पर विचार कीजिये:

    1. इसके तंतुओं को ऊन और कपास के तंतुओं के साथ, उनके गुणधर्मों को प्रबलित करने हेतु, सम्मिश्रित किया जा सकता है।
    2. इससे बने पात्रों को किसी भी मादक पेय को रखने के लिए उपयोग किया जा सकता है।
    3. इससे बनी बोतलों का पुनर्चक्रण (रीसाइक्लिंग) कर उनसे अन्य उत्पाद बनाए जा सकते हैं ।
    4. इससे बनी वस्तुओं का भस्मीकरण द्वारा, बिना ग्रीनहाउस गैस उत्सर्जन किए, आसानी से निपटान किया जा सकता है।

    उपर्युक्त कथनों में कौन-से सही हैं?

    (a) 1 और 3

    (b) 2 और 4

    (c) 1 और 4

    (d) 2 और 3

    उत्तर: (A) 

    व्याख्या:

    • PET (Polyethylene Terephthalate) पॉलिएस्टर का एक प्रकार है। 
    • इसका प्रयोग खाद्य पदार्थों और पेय पदार्थों आदि की पैकेजिंग के लिये प्लास्टिक की बोतलों और कंटेनर आदि बनाने में होता है। 
    • इसे खाद्य पैकेजिंग सामग्री के लिये उपयुक्त माना जाता है क्योंकि यह हल्का, गैर-प्रतिक्रियाशील, किफायती है।
    • ये प्रायः अन्य फाइबर जैसे रेयान, ऊन और कपास के साथ टिकाऊ-प्रेस मिश्रणों में उपयोग किये जाते हैं, जो इन तंतुओं के अंतर्निहित गुणों को मज़बूत करते हैं, जबकि कपड़े की सिकुड़न से उबरने की क्षमता में योगदान करते हैं। 
    • PET का सबसे व्यापक रूप से पुनर्नवीनीकृत किये जाने वाले PET बोतलें और कंटेनर हैं जिन्हें आमतौर पर पिघलाया जाता है और फाइबरफिल या कालीन के लिये रेशों/फाइबर के रूप में उपयोग किया जाता है। PET पुनर्नवीनीकरण करने के बाद उसको उस उद्देश्य हेतु पुनः उपयोग किया जा सकता है जिसके लिये उसे बनाया गया था, और PET में पुन: संश्लेषण के लिये बहुलक को उसके पूर्वगामी रासायनिक घटकों में तोड़ने के लिये तरीके तैयार किये गए हैं।

    अतः विकल्प (a) सही है।

    57. निम्नलिखित में से कौन-सा पक्षी नहीं है?

    (a) गोल्डन महासीर

    (b) इंडियन नाइटजार

    (c) स्पूनबिल

    (d) व्हाईट आइबिस

    उत्तर: (A)

    व्याख्या:

    • महसीर शब्द दो शब्दों से मिलकर बना है माही - मछली और शेर - बाघ, इसीलिये इसे मछलियों के बीच बाघ भी कहा जाता है। यह एक बड़ी साइप्रिनिड है और ताज़े जल की स्पोर्ट मछलियों में इसे सबसे कठोर माना जाता है। एक वयस्क गोल्डन महसीर के शरीर का रंग पृष्ठ की तरफ सुनहरा होता है और पंख लाल-पीले होते हैं। इसके अलावा बड़े शल्क और मोटे शक्तिशाली होंठों के साथ अपेक्षाकृत लंबे बार्बल्स (मुंह के सामने संवेदी बाल जैसे अंग) इसकी विशेषता हैं।
    • महसीर एक संवेदनशील प्रजाति है जो जिसके लिये जल पर्यावरण में परिवर्तन को सहन करना कठिन होता है। विश्व में मौजूद महसीर की 47 प्रजातियों में से पंद्रह प्रजातियाँ भारत में पाई जाती हैं।

    अतः विकल्प (a) सही है।

    58. निम्नलिखित में कौन-से, नाइट्रोजन यौगिकीकरण पादप हैं?

    1. अल्फाल्फा
    2. चौलाई (ऐमरंथ)
    3. चना (चिक-पी)
    4. तिपतिया घास (क्लोवर)
    5. कुलफा (पर्सलेन)
    6. पालक

    नीचे दिए कूट का प्रयोग कर सही उत्तर चुनिए:

    (a) केवल 1, 3 और 4

    (b) केवल 1, 3, 5 और 6

    (c) केवल 2, 4, 5 और 6

    (d) 1, 2, 4, 5 और 6

    उत्तर:(A) 

    व्याख्या:

    • नाइट्रोजन स्थिरीकरण कोई भी प्राकृतिक या औद्योगिक प्रक्रिया है जो नाइट्रोजन (N2) के मुक्त होने का कारण बनती है। 
    • नाइट्रोजन हवा में मौजूद एक अपेक्षाकृत अक्रिय गैस है, जो रासायनिक रूप से अन्य तत्त्वों के साथ मिलकर अधिक प्रतिक्रियाशील नाइट्रोजन यौगिकों जैसे कि अमोनिया, नाइट्रेट्स अथवा नाइट्राइट का निर्माण करती है।
    • दो प्रकार के नाइट्रोजन-स्थिरीकरण सूक्ष्मजीवों को मान्यता दी जाती है: मुक्त-जीवित (गैर-सहजीवी) बैक्टीरिया, जिसमें सायनोबैक्टीरिया (या नील-हरित शैवाल) एनाबेना और नोस्टोकैंड सामान्य जैसे एज़ोटोबैक्टर (Azotobacter), बेजरिनकिया (Beijerinckia) और क्लोस्ट्रीडियम (Clostridium) शामिल हैं; और पारस्परिक (सहजीवी) बैक्टीरिया जैसे राइजोबियम (Rhizobium), फलीदार पौधों से जुड़े, और विभिन्न एज़ोस्पिरिलम प्रजातियाँ, जो अनाज घास से संबंधित होती हैं।
    • उदाहरण: अल्फाल्फा, बीन्स, तिपतिया घास (Clovers), मटर, सोयाबीन आदि।

    अतः विकल्प (a) सही है।

    59. निम्नलिखित स्थितियों में से किस एक में ‘‘जैवशैल प्रौद्योगिकी (बायोरॉक टेक्नोलॉजी)’’ की बातें होती हैं?

    (a) क्षतिग्रस्त प्रवाल भित्तियों (कोरल रीफ्स) की बहाली

    (b) पादप अवशिष्टों का प्रयोग कर भवन-निर्माण सामग्री का विकास

    (c) शेल गैस के अन्वेषण/निष्कर्षण के लिए क्षेत्रों की पहचान करना

    (d) वनों/संरक्षित क्षेत्रों में जंगली पशुओं के लिए लवण-लेहिकाएँ (साल्ट लिक्स) उपलब्ध कराना

    उत्तर: (A)

    व्याख्या:

    • बायोरॉक, इस्पात संरचनाओं पर निर्मित समुद्री जल में विलेय खनिजों के विद्युत संचय से बनने वाला पदार्थ है। इन इस्पात संरचनाओं को समुद्र के तल पर उतारा जाता है और सौर पैनलों की सहायता से इनको ऊर्जा प्रदान की जाती है जो समुद्र की सतह पर तैरते रहते हैं।
    • यह प्रौद्योगिकी पानी में इलेक्ट्रोड के माध्यम से विद्युत की एक छोटी मात्रा को प्रवाहित करने का काम करती है।
    • जब एक धनावेशित एनोड और ऋणावेशित कैथोड को समुद्र के तल पर रखकर उनके बीच विद्युत प्रवाहित की जाती है तो कैल्शियम आयन और कार्बोनेट आयन आपस में संयोजन करते हैं जिससे कैल्शियम कार्बोनेट का निर्माण होता है, कोरल लार्वा कैल्शियम कार्बोनेट की उपस्थिति में तेज़ी से बढ़ते हैं।

    https://www.drishtiias.com/hindi/daily-updates/daily-news-analysis/restoration-of-coral-reefs

    60. ‘‘मियावाकी पद्धति’’ किसके लिए विख्यात है?

    (a) शुष्क और अर्ध-शुष्क क्षेत्रों में वाणिज्यिक कृषि का संवर्धन

    (b) आनुवंशिकत: रूपांतरित पुष्पों का प्रयोग कर उद्यानों का विकास

    (c) शहरी क्षेत्रों में लघु वनों का सृजन

    (d) तटीय क्षेत्रों और समुद्री सतहों पर पवन ऊर्जा का संग्रहण

    उत्तर: (C)

    व्याख्या:

    • मियावाकी पद्यति के प्रणेता जापानी वनस्पति वैज्ञानिक अकीरा मियावाकी (Akira Miyawaki) हैं। इस पद्यति से बहुत कम समय में जंगलों को घने जंगलों में परिवर्तित किया जा सकता है।
    • इस योजना ने घरों के आगे अथवा पीछे खाली पड़े स्थान (Backyards) को छोटे बागानों में बदलकर शहरी वनीकरण की अवधारणा में क्रांति ला दी है। इस पद्धति में देशी प्रजाति के पौधे एक दूसरे के समीप लगाए जाते हैं, जो कम स्थान घेरने के साथ ही अन्य पौधों की वृद्धि में भी सहायक होते हैं। सघनता की वज़ह से ये पौधे सूर्य की रौशनी को धरती पर आने से रोकते हैं, जिससे धरती पर खरपतवार नहीं उग पाता है। तीन वर्षों के पश्चात् इन पौधों को देखभाल की आवश्यकता नही होती है।
    • पौधे की वृद्धि 10 गुना तेज़ी से होती है जिसके परिणामस्वरूप वृक्षारोपण सामान्य स्थिति से 30 गुना अधिक सघन होता है।
    • जंगलों को पारंपरिक विधि से उगने में लगभग 200 से 300 वर्षों का समय लगता है, जबकि मियावाकी पद्धति से उन्हें केवल 20 से 30 वर्षों में ही उगाया जा सकता है।

    https://www.drishtiias.com/hindi/daily-updates/daily-news-analysis/a-japanese-touch-to-telangana-green-drive

    61. निम्नलिखित पर विचार कीजिये:

    1. आरोग्य सेतु
    2. कोविन
    3. डिजीलॉकर
    4. दीक्षा

    उपर्युक्त में से कौन-से, ओपेन सोर्स डिजिटल प्लेटफॉर्म पर बनाए गए हैं?

    (a)  केवल 1 और 2

    (b)  केवल 2, 3 और 4

    (c)  केवल 1, 3 और 4

    (d)  1, 2, 3 और 4

    उत्तर. D

    व्याख्या:

    • इस आरोग्य सेतु की प्रमुख खासियत पारदर्शिता, निजता तथा सुरक्षा रहा है और भारत की ओपन सोर्स सॉफ्टवेयर नीति की तर्ज पर आरोग्य सेतु के सोर्स कोड को अब सार्वजनिक कर दिया गया है।
    • CoWin प्लेटफॉर्म को ओपन सोर्स बनाया जा रहा है, जो सभी लोगों और  देशों के लिये उपलब्ध है।
    • दीक्षा – स्कूली शिक्षा के लिये डिजिटल इंफ्रास्ट्रक्चर भारत सरकार के शिक्षा मंत्रालय के तहत राष्ट्रीय शैक्षिक अनुसंधान और प्रशिक्षण परिषद की एक पहल है। 
      • दीक्षा पोर्टल 5 सितंबर, 2017 को भारत के उपराष्ट्रपति द्वारा लॉन्च किया गया था। यह एक ओपन सोर्स प्लेटफार्म है जो देश भर के सभी राज्यों और केंद्रशासित प्रदेशों के लिये उपलब्ध है। यह शिक्षकों, छात्रों और अभिभावकों के लिये राष्ट्रीय स्तर पर एक सामान्य डिजिटल प्लेटफॉर्म है। यह शिक्षकों को कक्षा संसाधनों, मूल्यांकन सहायता, समाचार और घोषणाओं, शिक्षक समुदाय आदि में प्रशिक्षण सामग्री तक पहुँच बनाने में सक्षम बनाता है। यह शिक्षक शिक्षा और प्रशिक्षण के लिये एक मुख्य डिजिटल तंत्र है। यह छात्रों को पाठ्यपुस्तक क्यूआर कोड को स्कैन करके अध्ययन सामग्री तक पहुँच प्राप्त करने में सक्षम बनाता है। नई शिक्षा नीति (एनईपी) को एक वर्ष पूरा होने के अवसर पर, प्रधान मंत्री ने शिक्षा प्रणाली के परिवर्तन में योगदान देने वाले विभिन्न सुधारों की शुरुआत की। लॉन्च के दौरान पीएम ने यह भी कहा कि दीक्षा पोर्टल जो कि सरकार का ई-लर्निंग पोर्टल है, दीक्षा को सनबर्ड ईडी का उपयोग करके बनाया गया है, जो पूरी तरह कार्यात्मक समाधान बिल्डिंग ब्लॉक है जो एमआईटी लाइसेंस के तहत ओपन-सोर्स सॉफ्टवेयर के रूप में उपलब्ध है और सनबर्ड का हिस्सा है, जो "मेड इन इंडिया, मेड फॉर द वर्ल्ड" डिजिटल पब्लिक गुड (DPG) है। 
    • डिजिलॉकर डिजिटल इंडिया के तहत एक महत्त्वपूर्ण पहल है, भारत सरकार के इस प्रमुख कार्यक्रम का उद्देश्य भारत को डिजिटल रूप से सशक्त समाज और ज्ञान अर्थव्यवस्था में बदलना है। कागज रहित शासन के विचार पर लक्षित, डिजिलॉकर डिजिटल तरीके से दस्तावेजों और प्रमाणपत्रों को जारी करने और सत्यापन के लिये एक मंच है, इस प्रकार यह भौतिक दस्तावेजों के उपयोग को समाप्त करता है।

    अतः विकल्प (d) सही है।

    62. वेब 3.0 के सन्दर्भ में, निम्नलिखित कथनों पर विचार कीजिये:

    1. वेब 3.0 प्रौद्योगिकी से व्यक्ति अपने स्वयं के आंकड़ों पर नियंत्रण कर सकते हैं।
    2. वेब 3.0 संसार में, ब्लॉकचेन आधारित सामाजिक नेटवर्क हो सकते हैं।
    3. वेब 3.0 किसी निगम द्वारा परिचालित होने की बजाय प्रयोक्ताओं द्वारा सामूहिक रूप से परिचालित किया जाता है।

    उपर्युक्त कथनों में कौन-से सही हैं?

    (a)  केवल 1 और 2

    (b) केवल 2 और 3

    (c)  केवल 1 और 3

    (d) 1, 2 और 3

    उत्तर: d

    व्याख्या:

    • वेब 3 की अवधारणा, जिसे वेब 3.0 भी कहा जाता है, का उपयोग इंटरनेट के संभावित अगले चरण का वर्णन करने के लिये किया जाता है और यह वर्ष 2021 में काफी चर्चा में रहा है।
    • वेब 3.0 एक विकेंद्रीकृत  इंटरनेट है जो ब्लॉकचेन तकनीक पर आधारित है यह उपयोग में आने वाले संस्करणों, वेब 1.0 और वेब 2.0 से अलग होगा।
    • वेब 3 में उपयोगकर्त्ताओं के पास प्लेटफॉर्म और एप्लीकेशन में स्वामित्व हिस्सेदारी होगी जो तकनीकी प्लेटफॉर्म को नियंत्रित करते हैं।
    • ब्लॉक चेन टेक्नोलॉजी कंपनी एथेरियम ( Ethereum) के संस्थापक गेविन वुड ने वर्ष 2014 में पहली बार वेब 3 शब्द का इस्तेमाल किया था और पिछले कुछ वर्षों में कई अन्य लोगों ने वेब 3 के विचार को जोड़ा है।

    वेब 3.0 का महत्त्व:

    • विकेंद्रीकृत और निष्पक्ष इंटरनेट: वेब 3.0 एक विकेंद्रीकृत और निष्पक्ष इंटरनेट प्रदान करेगा, जहाँ उपयोगकर्त्ता अपने स्वयं के डेटा को नियंत्रित कर सकते हैं।
    • यह मध्यस्थों को हटाता है: ब्लॉकचेन के साथ लेन-देन का समय और स्थान स्थायी रूप से दर्ज किया जाता है।
      • इस प्रकार वेब 3 मध्यस्थ की भूमिका को समाप्त कर सहकर्मी से सहकर्मी (विक्रेता से खरीदार) के मध्य लेन-देन को बढ़ावा देता है। इस अवधारणा को निम्नलिखित प्रकार से बढ़ाया जा सकता है
    • विकेंद्रीकरण और पारदर्शिता: वेब 3 विकेंद्रीकृत स्वायत्त संगठन (DAO) पर केंद्रित है।
      • DAO सभी व्यावसायिक नियमों से संबंधित है एवं किसी भी लेन-देन में शासी नियम किसी को भी देखने के लिये पारदर्शी रूप से उपलब्ध हैं तथा  इन नियमों के अनुरूप सॉफ्टवेयर के द्वारा लिखा जाएगा।
      • DAO के साथ प्रमाणित या मान्य करने के लिये केंद्रीय प्राधिकरण की कोई आवश्यकता नहीं है।
    • https://www.drishtiias.com/hindi/daily-updates/daily-news-analysis/web-3-0 

    63. ‘‘सॉफ्टवेयर, सेवा के रूप में "(Software as a Service (Saas)" के सन्दर्भ में, निम्नलिखित कथनों पर विचार कीजिये:

    1. SaaS क्रयकर्ता, प्रयोक्ता अन्तरापृष्ठ को अपनी आवश्यकतानुसार निर्धारित कर आंकड़ों के क्षेत्र में बदलाव कर सकते हैं।
    2. SaaS प्रयोक्ता, अपनी चल युक्तियों (मोबाइल डिवाइसेज़) के माध्यम से अपने आंकड़ों तक पहुँच बना सकते हैं।
    3. आउटलुक, हॉटमेल और याहू मेल SaaS के रूप हैं।

    उपर्युक्त कथनों में कौन से सही हैं?

    (a) केवल 1 और 2

    (b) केवल 2 और 3

    (c) केवल 1 और 3

    (d) 1, 2 और 3

    उत्तर: d

    व्याख्या:

    • SaaS सॉफ्टवेयर डिलीवरी का एक तरीका है जो इंटरनेट कनेक्शन और वेब ब्राउज़र के साथ किसी भी डिवाइस से डेटा एक्सेस करने की अनुमति देता है। इस वेब-आधारित मॉडल में, सॉफ्टवेयर विक्रेता सर्वर, डेटाबेस और एप्लिकेशन को बनाने वाले कोड की होस्टिंग और रख-रखाव करते हैं।
    • SaaS उपयोग का सामान्य रूप एक वेब-आधारित ईमेल सेवा है जैसे आउटलुक, हॉटमेल या याहू! मेल, इन सेवाओं के साथ आप अपने एकाउंट को इंटरनेट पर प्रायः वेब ब्राउज़र से एक्सेस करते हैं।

    64. निम्नलिखित कथनों में कौन-सा एक, जनसंचार-माध्यमों में बहुचर्चित ‘‘प्रभाजी कक्षीय बमबारी प्रणाली’’ के आधारभूत विचार को सर्वोत्तम रूप से प्रतिबिंबित करता है?

    (a) अंतरिक्ष में अतिध्वनिक मिसाइल का प्रमोचन, पृथ्वी की तरफ बढ़ते हुए क्षुद्रग्रह का सामना कर उसका अंतरिक्ष में ही विस्फोटन कराने के लिए किया जाता है।

    (b) कोई अंतरिक्षयान अनेक कक्षीय गतियों के बाद किसी अन्य ग्रह पर उतरता है।

    (c) कोई मिसाइल पृथ्वी के परित: किसी स्थिर कक्षा में स्थापित किया जाता है और वह पृथ्वी पर किसी लक्ष्य के ऊपर कक्षा को त्यागता है।

    (d) कोई अंतरिक्षयान किसी धूमकेतु के साथ-साथ उसी चाल के चलते हुए उसके पृष्ठ पर एक संपरीक्षित्र स्थापित करता है।

    उत्तर: c

    व्याख्या:

    • फ्रैक्शनल ऑर्बिटल बॉम्बार्डमेंट सिस्टम (FOBS) वारहेड डिलीवरी सिस्टम है जो अपने लक्ष्य गंतव्य की ओर लो अर्थ ऑर्बिट का उपयोग करता है। लक्ष्य तक पहुँचने से ठीक पहले यह एक प्रतिगामी इंजन बर्न से होकर गुज़रता है।
    • सोवियत संघ ने पहली बार 1960 के दशक में FOBS को परमाणु-हथियार वितरण प्रणाली के रूप में विकसित किया था। यह परमाणु हथियार भेजने  के लिये अंतरिक्ष का उपयोग करने वाले पहले सोवियत प्रयासों में से एक था।

    65. ‘‘क्यूबिट (qubit)’’ शब्द का उल्लेख निम्नलिखित में कौन-से एक प्रसंग में होता है?

    (a)  क्लाउड सेवाएँ

    (b)  क्वांटम संगणन

    (c)  दृश्य प्रकाश संचार प्रौद्योगिकियाँ

    (d)  बेतार संचार प्रौद्योगिकियाँ

    उत्तर: B

    व्याख्या:

    • क्वांटम कंप्यूटर 'क्यूबिट्स' (या क्वांटम बिट्स) में गणना करते हैं। वे क्वांटम यांत्रिकी के गुणों का उपयोग करते हैं। क्वांटम यांत्रिकी वह विज्ञान जो परमाणु पैमाने पर पदार्थ के व्यवहार को नियंत्रित करता है।

    66. निम्नलिखित संचार प्रौद्योगिकियों पर विचार कीजिये:

    1. निकट-परिपथ (क्लोज़-सर्किट) टेलीविज़न
    2. रेडियो आवृत्ति अभिनिर्धारण
    3. बेतार स्थानीय क्षेत्र नेटवर्क

    उपर्युक्त में कौन-सी लघु-परास युक्तियाँ/प्रौद्योगिकियाँ मानी जाती हैं?

    (a)  केवल 1 और 2

    (b)  केवल 2 और 3

    (c)  केवल 1 और 3

    (d)  1, 2 और 3

    उत्तर: d

    व्याख्या:

    • शॉर्ट रेंज डिवाइस (SRD) ऐसे रेडियो उपकरण हैं जो अन्य रेडियो सेवाओं के साथ व्यवधान (Interference) का कम जोखिम प्रदान करते हैं, क्योंकि उनकी संचार शक्ति और सीमा कम होती है।  'शॉर्ट रेंज डिवाइस' कई अलग-अलग प्रकार के वायरलेस उपकरणों पर प्रयोग की जा सकती है, जिनमें शामिल हैं:
      • अभिगम नियंत्रण
      • अलार्म और मूवमेंट डिटेक्टर
      • क्लोज्ड-सर्किट टेलीविज़न (CCTV)
      • वायरलेस माइक्रोफोन सहित ताररहित ऑडियो उपकरण
      • औद्योगिक नियंत्रण
      • स्थानीय क्षेत्र नेटवर्क
      • चिकित्सा प्रत्यारोपण
      • मीटरिंग डिवाइस
      • रिमोट कंट्रोल
      • रेडियो फ्रीक्वेंसी आइडेंटिफिकेशन (RFID)
      • सड़क परिवहन टेलीमैटिक्स
      • टेलीमेट्री।

    https://www.etsi.org/technologies/short-range-devices

    67. निम्नलिखित कथनों पर विचार कीजिये:

    1. जैवपरत (बायोफिल्म) मानव ऊतकों के भीतर चिकित्सकीय अंतर्रोपों पर बन सकती हैं।
    2. जैवपरत खाद्यपदार्थ और खाद्य प्रसंस्करण सतहों पर बन सकती हैं।
    3. जैवपरत प्रतिजैविक प्रतिरोध दर्शा सकती हैं।

    उपर्युक्त कथनों में कौन-से सही हैं?

    (a) केवल 1 और 2

    (b)  केवल 2 और 3

    (c) केवल 1 और 3

    (d) 1, 2 और 3

    उत्तर: D

    व्याख्या:

    • चिकित्सा की भाषा में जैवपरत चिकित्सकीय अंतर्रोपो और मानव ऊतक के भीतर सिस्टिक फाइब्रोसिस के रूप में बनते हैं, उद्योग में ये उपकरण की सतह को कवर करते हैं। जैवपरत अपने इनहैबीटेंट को विभिन्न प्रतिकूल पर्यावरणीय परिस्थितियों (बायोसाइड्स और एंटीबायोटिक) से बचाते हैं।
    • खाद्य उद्योग के परिवेश में जैव परत के निर्माण तेज़ी से हो सकता है। जैवपरत का निर्माण तीन मुख्य घटकों के बीच परस्पर क्रिया पर निर्भर करता है: जीवाणु कोशिकाएँ, इनके बनने की सतह और आसपास का माध्यम।
    • जैवपरत के भीतर माइक्रोबियल कोशिकाओं ने प्लवक की कोशिकाओं की तुलना में 10-1000 गुना अधिक एंटीबायोटिक प्रतिरोध प्रदर्शित किया है।

    https://onlinelibrary.wiley.com/doi/pdf/10.1002/jppr2002322153

    https://www.livescience.com/57295-biofilms.html 

    68. प्रजैविकों (प्रोबायोटिक्स) के सन्दर्भ में निम्नलिखित कथनों पर विचार कीजिये:

    1. प्रजैविक, जीवाणु और यीस्ट दोनों के बने होते हैं।
    2. प्रजैविकों में जीव, खाए जाने वाले खाद्य में होते हैं किन्तु वे नैसर्गिक रूप से हमारी आहार-नली में नहीं पाए जाते।
    3. प्रजैविक दुग्ध शर्कराओं के पाचन में सहायक हैं।

    उपर्युक्त कथनों में कौन-सा/से सही है/हैं?

    (a)  केवल 1

    (b)  केवल 2

    (c)  1 और 3

    (d)  2 और 3

    उत्तर: c

    व्याख्या:

    • प्रोबायोटिक्स जीवित लाभकारी बैक्टीरिया और/या यीस्ट का एक संयोजन हैं जो स्वाभाविक रूप से शरीर के भीतर रहते हैं। बैक्टीरिया को आमतौर पर नकारात्मक दृष्टि से ऐसे जीवों के रूप में देखा जाता है जो आपको बीमार बनाते हैं।
    • एसिडोफिलस एक प्रोबायोटिक बैक्टीरिया है जो स्वाभाविक रूप से मानव आंत्र और शरीर के अन्य भागों में उपस्थित होता है। यह बैक्टीरिया पाचन तंत्र को लैक्टोज जैसी शर्करा को लैक्टिक एसिड में विखंडित करने में मदद करता है। मानव आंत्र में खरबों बैक्टीरिया और अन्य सूक्ष्म जीव उपस्थित होते हैं। 
    • ऐसे कई तरीके हैं जिनसे मानव प्रोबायोटिक सप्लीमेंट ले सकते हैं।ये कई रूपों में आते हैं, जिनमें खाद्य पदार्थ, पेय, कैप्सूल या गोलियांँ, पाउडर, तरल पदार्थ शामिल हैं।

    अतः कथन 2 और 3 सही है।

    https://www.healthline.com/health/probiotics-and-digestive-health

    69. कोविड-19 विश्वमहामारी को रोकने के लिए बनाई जा रही वैक्सीनों के प्रसंग में, निम्नलिखित कथनों पर विचार कीजिये:

    1. भारतीय सीरम संस्थान ने mRNA प्लेटफॉर्म का प्रयोग कर कोविशील्ड नामक कोविड-19 वैक्सीन निर्मित की।
    2. स्पुतनिक ट वैक्सीन रोगवाहक (वेक्टर) आधारित प्लेटफॉर्म का प्रयोग कर बनाई गई है।
    3. कोवैक्सीन एक निष्कृत रोगजनक आधारित वैक्सीन है।

    उपर्युक्त कथनों में कौन-से सही हैं?

    (a)  केवल 1 और 2

    (b)  केवल 2 और 3

    (c)  केवल 1 और 3

    (d)  1, 2 और 3

    उत्तर: B 

    व्याख्या: 

    • COVISHIELD वैक्सीन उस प्लेटफॉर्म पर आधारित है जो SARS-CoV-2 स्पाइक (S) ग्लाइकोप्रोटीन को एन्कोडिंग करने वाले एक पुनःसंयोजक, प्रतिकृति-रहित चिंपैंजी एडेनोवायरस वेक्टर का उपयोग करता है। इसे लगाए जाने के बाद, कोरोनावायरस के हिस्से की आनुवंशिक सामग्री प्रकट होती है जो एक प्रतिरक्षा प्रतिक्रिया को उत्तेजित करती है। अतः कथन 1 सही नहीं है।
    • स्पुतनिक V एक अच्छी तरह से अध्ययन किये गए मानव एडेनोवायरस वेक्टर प्लेटफॉर्म पर आधारित विश्व का पहला पंजीकृत टीका है। इसे 4 अरब लोगों की कुल आबादी वाले 71 देशों में उपयोग के लिये मंज़ूरी दी गई है। वैक्सीन का नाम पहले सोवियत अंतरिक्ष उपग्रह के नाम पर रखा गया है। 5 दिसंबर, 2020 और 31 मार्च, 2021 के बीच दोनों वैक्सीन घटकों के साथ टीका लगाए गए रूसियों के बीच कोरोनावायरस की घटनाओं के आँकड़ों के विश्लेषण के आधार पर, वैक्सीन की प्रभावकारिता 97.6% है। अतः कथन 2 सही है।
    • Covaxin एक निष्क्रिय वायरल टीका है। इस वैक्सीन को होल-विरियन इनएक्टिवेटेड वेरो सेल-व्युत्पन्न तकनीक से विकसित किया गया है। उनमें निष्क्रिय वायरस होते हैं, जो किसी व्यक्ति को संक्रमित नहीं कर सकते हैं, लेकिन फिर भी प्रतिरक्षा प्रणाली को सक्रिय वायरस के खिलाफ एक रक्षा तंत्र तैयार करना सिखा सकते हैं। कथन 3 सही है।

    https://www.who.int/publications/m/item/covaxin-(bbv152)-inactivated-covid-19-vaccine

    70. यदि कोई मुख्य सौर तूफऩ (सौर प्रज्वाल) पृथ्वी पर पहुँचता है, तो पृथ्वी पर निम्नलिखित में कौन-से संभव प्रभाव होंगे?

    1. GPS और दिक्संचालन (नैविगेशन) प्रणालियाँ विफल हो सकती हैं।
    2. विषुवतीय क्षेत्रों में सुनामियाँ आ सकती हैं।
    3. बिजली ग्रिड क्षतिग्रस्त हो सकते हैं।
    4. पृथ्वी के अधिकांश हिस्से पर तीव्र ध्रुवीय ज्योतियाँ घटित हो सकती हैं।
    5. ग्रह के अधिकांश हिस्से पर दावाग्नियाँ घटित हो सकती हैं ।
    6. उपग्रहों की कक्षाएँ विक्षुब्ध हो सकती हैं।
    7. ध्रुवीय क्षेत्रों के ऊपर से उड़ते हुए वायुयान का लघुतरंग रेडियो संचार बाधित हो सकता है।

    नीचे दिए कूट का प्रयोग कर सही उत्तर चुनिए:

    (a) केवल 1, 2, 4 और 5

    (b)  केवल 2, 3, 5, 6 और 7

    (c)  केवल 1, 3, 4, 6 और 7

    (d)  1, 2, 3, 4, 5, 6 और 7

    उत्तर:c

    व्याख्या:

    • 10 मई, 2022 को सूर्य ने एक मजबूत सौर तूफ़ान/सौर प्रज्ज्वाल उत्सर्जित किया, जो EDT पर सुबह 9:55 बजे चरम पर था। नासा की सोलर डायनेमिक्स ऑब्जर्वेटरी, जो लगातार सूर्य की स्थिति का अवलोकन करती है, ने इस घटना की एक छवि कैप्चर की। सौर प्रज्ज्वाल ऊर्जा के शक्तिशाली विस्फोट हैं। 
    • रात के आकाश में अक्सर तूफानों को सुंदर ज्योति के रूप में देखा जा सकता है, लेकिन वे पृथ्वी पर बिजली के ग्रिडों और नेविगेशन सिस्टम को क्षतिग्रस्त कर सकते हैं।
    • सूर्य की सतह से एक विशाल  सौर तूफ़ान (सौर प्रज्ज्वाल) उत्पन्न हुआ जिसने एक तीव्र चुंबकीय तूफान उत्पन्न करते हुए, रेडियो तरंगों, दूरसंचार नेटवर्क और बिजली प्रणालियों को बाधित किया।
    • वैज्ञानिकों ने ग्रीनलैंड और अंटार्कटिका से बर्फ के कोर के विश्लेषण के माध्यम से पृथ्वी की बर्फ के भीतर एक अत्यधिक सौर 'सुनामी' के प्रमाण प्राप्त किये हैं।
    • बड़े पैमाने पर प्लाज्मा के भंडारण के बाद एक चुंबकीय बांध बनता है। सौर चक्र के अंत में, यह चुंबकीय बांध टूट सकता है और ध्रुवों की ओर सुनामी की तरह भारी मात्रा में प्लाज्मा कैस्केडिंग मुक्त करता है।

    अतः कथन 1, 3, 4, 6 और 7 सही हैं।

    https://www.inverse.com/science/a-recent-uptick-in-solar-storms-has-scientists-worried 

    https://www.wionews.com/science/scientists-find-evidence-of-an-extreme-solar-tsunami-deep-within-earths-ice-448784 

    https://www.drishtiias.com/daily-updates/daily-news-analysis/solar-tsunami-can-trigger-the-sunspot-cycle

    71. निम्नलिखित फस़लों में कौन-सी एक, मेथैन और नाइट्रस ऑक्साइड दोनों का सर्वाधिक महत्त्वपूर्ण मानवोद्भवी स्रोत है?

    (a) कपास

    (b) धान

    (c) गन्ना

    (d) गेहूँ

    उत्तर: (b) 

    व्याख्या:

    • धान, मेथैन और नाइट्रस ऑक्साइड दोनों का सर्वाधिक महत्त्वपूर्ण मानवोद्भवी स्रोत है। वर्तमान में धान का उत्पादन बड़ी चुनौतियों का सामना कर रहा है जिसमें सिंचाई के पानी की कमी और जारी जलवायु परिवर्तन शामिल हैं। 
    • वर्तमान फसल तकनीकों के संशोधन से उपज में वृद्धि हो सकती है तथा पानी की बचत हो सकती है और ग्रीनहाउस गैस उत्सर्जन को भी कम किया जा सकता है।

    अतः विकल्प (c) सही है।

    72. कृषि की ‘‘धान गहनता प्रणाली’’ का, जिसमें धान के खेतों का बारी-बारी से क्लेदन और शुष्कन किया जाता है, क्या परिणाम होता है?

    1. बीज की कम आवश्यकता
    2. मेथैन का कम उत्पादन
    3. बिजली की कम खपत

    नीचे दिए कूट का प्रयोग कर सही उत्तर चुनिए:

    (a)  केवल 1 और 2

    (b)  केवल 2 और 3

    (c)  केवल 1 और 3

    (d)  1, 2 और 3

    उत्तर: (d) 

    व्याख्या:

    • कृषि की ‘‘धान गहनता प्रणाली’’ का, जिसमें धान के खेतों का बारी-बारी से क्लेदन और शुष्कन किया जाता है, इस पद्धति का मुख्य परिणाम बीज की कम आवश्यकता, मेथैन का कम उत्पादन एवं बिजली की कम खपत है।

    अतः विकल्प (d) सही है।

    https://hindi.indiawaterportal.org/content/esaaraai-takanaika-nae-badhaayaa-caavala-kaa-utapaadana/content-type-page/47948

    73. पश्चिम अफ्रीका की निम्नलिखित झीलों में कौन-सी एक, सूखकर मरुस्थल में बदल गई है?

    (a)  लेक विक्टोरिया

    (b)  लेक फागुबिन

    (c)  लेक ओगुटा

    (d)  लेक वोल्टा

    उत्तर: (b) 

    व्याख्या:

    • संयुक्त राष्ट्र जलवायु परिवर्तन सम्मेलन-COP 26 के अनुसार पश्चिम अफ्रीका की लेक फागुबिन (Lake Faguibine) झील सूखकर मरुस्थल में बदल गई है।

    अतः विकल्प (c) सही है।

    74. दक्षिण भारत की गंडिकोटा घाटी (कैन्यन) निम्नलिखित नदियों में से किस एक से निर्मित हुई है?

    (a) कावेरी

    (b) मंजिरा

    (c) पेन्नार

    (d) तुंगभद्रा

    उत्तर: c

    व्याख्या: 

    • गंडिकोटा आंध्र प्रदेश के कडप्पा ज़िले का एक ट्वी गाँव है, जिसे ‘भारत की मत्य घाटी’ के रूप में जाना जाता है। दाहिनी ओर पेन्नार नदी से घिरा, यह गाँव एर्रामला पहाड़ियों से होकर नदी के द्वारा बनाई गई शानदार घाटियों के लिये लोकप्रिय है। उनके बीच चलने वाली धाराओं के साथ संकरी घाटियाँ और खड़ी चट्टानी दीवारें बेहद ही खूबसूरत लगती हैं।

    75. निम्नलिखित युग्मों पर विचार कीजिये:

    शिखर पर्वत
    1. नामचा बरवा गढ़वाल हिमालय
    2. नंदा देवी कुमाऊँ हिमालय
    3. नोकरेक सिक्किम हिमालय

    उपर्युक्त युग्मों में कौन-सा/से सही सुमेलित है/हैं?

    (a) 1 और 2

    (b) केवल 2

    (c) 1 और 3

    (d) केवल 3

    उत्तर: (b)

    व्याख्या:

     नंदादेवी चोटियाँ कुमाऊँ हिमालय का एक भाग हैं।

    • सतलुज और काली नदियों के बीच स्थित हिमालय के हिस्से को कुमाउँ हिमालय के नाम से जाना जाता है।
    • वृहद् हिमालय का विस्तार पश्चिम में नंगा पर्वत से लेकर पूर्व में नामचा बरवा चोटी तक विस्तृत है।
    • नामचा बरवा पर्वत श्रेणी तिब्बत के अंतर्गत आती है।

    76. अक्सर समाचारों में सुनाई देने वाला शब्द ‘‘लिवैंट’’ मोटे तौर पर निम्नलिखित में से किस क्षेत्र से संगत है?

    (a)  पूर्वी भूमध्यसागरीय तट के पास का क्षेत्र

    (b)  उत्तरी अफ्रीकी तट के पास का मिस्र से मोरक्को तक फैला क्षेत्र

    (c)  फारस की खाड़ी और अफ्रीका के  शृंग (हॉर्न ऑफफ़्रीका) के पास का क्षेत्र

    (d) भूमध्य सागर के सम्पूर्ण तटवर्ती क्षेत्र

    उत्तर. (a)

    व्याख्या:

    • लिवैंट भूमध्य सागर का पूर्वी तट है, जो लगभग 800 किमी. लंबा और लगभग 150 किमी चौड़ा है। यह पश्चिम में भूमध्य सागर और पूर्व में अरबो-सीरियाई रेगिस्तान के बीच में फैला हुआ है, जो उत्तर में ओरोंटिस नदी के मुहाने से लेकर दक्षिण में इस्थमस ऑफ़ स्वेज तक फैला हुआ है।

    अतः विकल्प (a) सही है।

    https://www.sciencedirect.com/topics/social-sciences/levant 

    77. निम्नलिखित देशों पर विचार कीजिये:

    1. अज़रबैजान
    2. किरगिज़स्तान
    3. ताजिकिस्तान
    4. तुर्कमेनिस्तान
    5. उज़्बेकिस्तान

    उपर्युक्त में से किनकी सीमाएँ अफगानिस्तान के साथ लगती हैं?

    (a) केवल 1, 2 और 5

    (b)  केवल 1, 2 3 और 4

    (c)  केवल 3, 4 और 5

    (d)  1, 2, 3, 4 और 5

    उत्तर. (C)

    व्याख्या:

    • अफगानिस्तान को मूल रूप से एरियाना या बैक्ट्रा के रूप में जाना जाता है और फिर खुरासान (उगते सूरज की भूमि) के रूप में जाना जाता है। यह पश्चिम में ईरान और तुर्कमेनिस्तान, उत्तर में उज्बेकिस्तान, ताजिकिस्तान और किर्गिस्तान, पूर्व और दक्षिण में पाकिस्तान तथा उत्तर-पूर्व में वखान प्रांत चीन द्वारा घिरा हुआ है।
    • अफगानिस्तान जिन देशों के साथ सीमा साझा करता है उनमें शामिल हैं: चीन, ईरान, पाकिस्तान, ताजिकिस्तान, तुर्कमेनिस्तान और उज़्बेकिस्तान

    Afghanistan

    अतः विकल्प (c) सही है

    http://saarc-sdmc.org/afghanistan

    78. भारत के सन्दर्भ में, निम्नलिखित कथनों पर विचार कीजिये:

    1. मोनाज़ाइट दुर्लभ मृदाओं का स्रोत है।
    2. मोनाज़ाइट में थोरियम होता है।
    3. भारत की समस्त तटवर्ती बालुकाओं में मोनाज़ाइट प्राकृतिक रूप में होता है।
    4. भारत में, केवल सरकारी निकाय ही मोनाज़ाइट संसाधित या निर्यात कर सकते हैं।

    उपर्युक्त कथनों में कौन-से सही हैं?

    (a) केवल 1, 2 और 3

    (b) केवल 1, 2 और 4

    (c) केवल 3 और 4

    (d) 1, 2, 3 और 4

    उत्तर. (b)

    व्याख्या:

    • मोनाजाइट अयस्क भारत, मेडागास्कर और दक्षिण अफ्रीका में पाया जाता है।
    • मोनाजाइट दुर्लभ मृदा तत्त्वों थोरियम, लैंथेनम और सेरियम का एक महत्त्वपूर्ण अयस्क है।

    मोनाजाइट के राज्यवार स्रोत:-

    राज्य

    मोनाजाइट  (मिलियन टन)

    ओडिशा

    2.41

    आंध्र प्रदेश

    3.72

    तमिलनाडु

    2.46

    केरल

    1.90

    पश्चिम बंगाल

    1.22

    झारखंड

    0.22

    कुल

    11.93

    • यह भारत के समस्त तटवर्ती क्षेत्रों में नहीं पाया जाता है।
    • वर्ष 1933 तक निजी कंपनियों को समुद्र तट तट की बालुकाओं में खनिजों का उत्खनन करने की अनुमति नहीं थी। उदारीकरण के बाद, निजी कंपनियों को शुरू में गार्नेट और सिलीमेनाइट की खान में खनन करने की अनुमति दी गई थी और इसके बाद अन्य खनिजों के लिये अनुमति दी गई थी।
      • वर्ष 2016 में पहले के एक संशोधन ने निजी कंपनियों को समुद्र तट की रेत के खनन पर रोक लगा दी, जहाँ  मोनाजाइट की संकेंद्रण 0.75% से अधिक था।
      • निजी फर्मों को मोनाजाइट के प्रसंस्करण या निर्यात से प्रतिबंधित किया गया है। यह एक सरकारी एकाधिकार बना हुआ है, जिसे परमाणु ऊर्जा विभाग के दायरे में रखा गया है।

    अतः विकल्प (b) सही है।

    https://science.thewire.in/politics/government/centre-bars-private-players-from-mining-beach-sand-minerals/

    79. उत्तरी गोलार्ध में, वर्ष का सबसे लंबा दिन आम तौर पर कब होता है?

    (a) जून महीने का पहला पखवाड़ा

    (b) जून महीने का दूसरा पखवाड़ा

    (c) जुलाई महीने का पहला पखवाड़ा

    (d) जुलाई महीने का दूसरा पखवाड़ा

    उत्तर. (b.)

    • उत्तरी गोलार्द्ध में '(21 जून)’ यानी जून महीने का दूसरा पखवाड़ा' साल का सबसे लंबा दिन होता है।

    अतः विकल्प (b) सही है।

    https://www.drishtiias.com/hindi/daily-updates/daily-news-analysis/the-tilt-or-why-thursday-had-longer-daylight-hours-than-any-other-day

    80. निम्नलिखित युग्मों पर विचार कीजिये:

    आर्द्रभूमि/झील अवस्थान
    1. होकेरा आर्द्रभूमि  पंजाब
    2. रेणुका आर्द्रभूमि हिमाचल प्रदेश
    3. रुद्रसागर झील त्रिपुरा
    4. सस्थाम्कोत्ता झील तमिलनाडु

    उपर्युक्त युग्मों में कितने सही सुमेलित हैं?

    (a) केवल एक युग्म 

    (b) केवल दो युग्म

    (c) केवल तीन युग्म

    (a) सभी चारों युग्म

    उत्तर. (b.)

    आर्द्रभूमियों/झीलों की सही अवस्थिति इस प्रकार है:

    आर्द्रभूमि/झील अवस्थिति
    1. होकेर आर्द्रभूमि जम्मू और कश्मीर
    2. रेणुका आर्द्रभूमि हिमाचल प्रदेश
    3. रुद्रसागर झील त्रिपुरा
    4. सस्थाम्कोत्ता झील केरल

    अतः विकल्प (b) सही है।

    https://www.drishtiias.com/hindi/to-the-points/paper3/wetlands-7 

    81. निम्नलिखित कथनों पर विचार कीजिये:

    1. एच.एन. सान्याल समिति की रिपोर्ट के अनुसरण में, न्यायालय की अवमानना अधिनियम, 1971 पारित किया गया था।
    2. भारत का संविधान उच्चतम न्यायालय और उच्च न्यायालयों को, अपनी अवमानना के लिए दंड देने हेतु, शक्ति प्रदान करता है।
    3. भारत का संविधान सिविल अवमानना और आपराधिक अवमानना को परिभाषित करता है।
    4. भारत में, न्यायालय की अवमानना के विषय में कानून बनाने के लिए संसद में शक्ति निहित है।

    उपर्युक्त कथनों में कौन-सा/से सही है/हैं? 

    (a) केवल 1 और 2

    (b) 1, 2 और 4

    (c) केवल 3 और 4

    (d)  केवल 3

    उत्तर: (b) 

    व्याख्या:

    • सत्यपाल समिति द्वारा तैयार किये गए 1963 के विधेयक का संसद की संयुक्त समिति (1969-70) (भार्गव समिति) द्वारा पुनर्विलोकन किया गया था। जिसके आधार पर न्यायालय की अवमानना अधिनियम, 1971 अधिनियमित किया गया था। अतः कथन 1 सही है
    • संविधान के अनुच्छेद 129 ने सर्वोच्च न्यायालय को स्वयं की अवमानना ​​के लिये दंडित करने की शक्ति प्रदान की। अनुच्छेद 215 ने उच्च न्यायालयों को संबंधित शक्ति प्रदान की। अतः कथन 2 सही है।
    • न्यायालय की अवमानना अधिनियम, 1971 (Contempt of Court Act, 1971) के अनुसार, न्यायालय की अवमानना का अर्थ किसी न्यायालय की गरिमा तथा उसके अधिकारों के प्रति अनादर प्रदर्शित करना है। अभिव्यक्ति 'अदालत की अवमानना' को संविधान द्वारा परिभाषित नहीं किया गया है। हालाँकि, संविधान के अनुच्छेद 129 ने सर्वोच्च न्यायालय को स्वयं की अवमानना ​​के लिये दंडित करने की शक्ति प्रदान की। अतः कथन 3 गलत है।
    • अवमानना के लिये दंड देने की शक्ति सर्वोच्च न्यायालय में निहित एक संवैधानिक शक्ति है जिसे विधायी अधिनियम द्वारा भी कम या समाप्त नहीं किया जा सकता है। अनुच्छेद 142 (2) में कहा गया है कि "संसद द्वारा इस संबंध में बनाए गए किसी भी कानून के प्रावधानों के अधीन" सर्वोच्च न्यायालय के पास अपनी अवमानना की सजा पर कोई भी आदेश देने की पूरी शक्ति होगी। अतः कथन 4 सत्य है।

    82. भारत के सन्दर्भ में, निम्नलिखित कथनों पर विचार कीजिये:

    1. सरकारी विधि अधिकारी और विधिक फर्म अधिवक्ता के रूप में मान्यता-प्राप्त हैं, किन्तु कॉर्पोरेट वकील और पेटेंट न्यायवादी अधिवक्ता की मान्यता से बाहर रखे गए हैं।
    2. विधिज्ञ परिषदों (बार कौंसिलों) को विधिक शिक्षा और विधि महाविद्यालयों की मान्यता के बारे में नियम अधिकथित करने की शक्ति है।

    उपर्युक्त कथनों में कौन-सा/से सही है/हैं?

    (a) केवल 1

    (b) केवल 2

    (c) 1 और 2 दोनों 

    (d) न तो 1, न ही 2

    उत्तर: (b) 

    व्याख्या:

    • सरकारी विधि अधिकारी, विधिक फर्म और पेटेंट न्यायवादी अधिवक्ता के रूप में मान्यता प्राप्त हैं, जबकि कॉर्पोरेट वकील अधिवक्ता की मान्यता से बाहर हैं। अतः कथन 1 सही नहीं है।
    • भारतीय विधिज्ञ परिषद, अधिवक्ता अधिनियम 1961 की धारा 4 के तहत स्थापित एक वैधानिक निकाय है जो भारत में विधिक अभ्यास और विधिक शिक्षा को नियंत्रित करता है। इसके सदस्य भारतीय वकीलों/अधिवक्ताओं में से चुने जाते हैं और इस प्रकार वे भारतीय विधिज्ञ परिषद का प्रतिनिधित्व करते हैं। यह पेशेवर आचरण, शिष्टाचार के मानकों को निर्धारित करता है और बार/विधिज्ञ परिषद पर अनुशासनात्मक अधिकार क्षेत्र का प्रयोग करता है। 
    • यह विधिक शिक्षा के लिये मानक भी निर्धारित करता है और उन विश्वविद्यालयों को मान्यता प्रदान करता है जिनकी विधि की डिग्री छात्रों के लिये स्नातक स्तर पर अधिवक्ता के रूप में नामांकन के लिये योग्यता के रूप में काम करेगी। अतः कथन 2 सही है।

    83. निम्नलिखित कथनों पर विचार कीजिये:

    1. किसी संविधान संशोधन विधेयक को भारत के राष्ट्रपति की पूर्व सिफारिश की अपेक्षा होती है।
    2. जब कोई संविधान संशोधन विधेयक भारत के राष्ट्रपति के समक्ष प्रस्तुत किया जाता है, तोे भारत के राष्ट्रपति के लिए यह बाध्यकर है कि वे अपनी अनुमति दें।
    3. संविधान संशोधन विधेयक लोक सभा और राज्य सभा दोनों द्वारा विशेष बहुमत से पारित होना ही चाहिए और इसके लिए संयुक्त बैठक का कोई उपबंध नहीं है।

    उपर्युक्त कथनों में कौन-से सही हैं?

    (a) केवल 1 और 2

    (b) केवल 2 और 3

    (c) केवल 1 और 3

    (d) 1, 2 और 3

    उत्तर: (b) 

    व्याख्या:

    • संविधान संशोधन विधेयक को किसी मंत्री या गैर सरकारी सदस्य द्वारा पुरःस्थापित किया जा सकता है और इसके लिये राष्ट्रपति की पूर्व सिफारिश आवश्यक नही है। अतः कथन 1 सही नही है
    • राष्ट्रपति विधेयक को सहमति/अनुमति देने के लिये बाध्य है। वह न तो विधेयक को अपने पास रख सकता है और न ही संसद के पास पुनर्विचार के लिये भेज सकता है। अतः कथन 2 सही है।
    • संविधान संसोधन विधेयक का प्रत्येक सदन में अलग-अलग पारित होना अनिवार्य है। दोनों सदनों के बीच असहमति की स्थिति में दोनों सदनों की संयुक्त बैठक का कोई उपबंध नही है। अतः कथन 3 सही है।

    84. निम्नलिखित कथनों पर विचार कीजिये:

    1. भारत का संविधान मंत्रियों को चार श्रेणियों, अर्थात् कैबिनेट मंत्री, स्वतंत्र प्रभार वाले राज्यमंत्री, राज्यमंत्री और उपमंत्री, में वर्गीकृत करता है।
    2. संघ सरकार में मंत्रियों की कुल संख्या, प्रधान मंत्री को मिला कर, लोक सभा के कुल सदस्याें के 15% से अधिक नहीं होनी चाहिए।

    उपर्युक्त कथनों में कौन-सा/से सही है/हैं?

    (a) केवल 1

    (b) केवल 2

    (c) 1 और 2 दोनों 

    (d) न तो 1, न ही 2

    उत्तर : (b)  

    व्याख्या:

    • मंत्रिपरिषद में मंत्रियों की तीन श्रेणियांँ होती है -कैबिनट मंत्री ,राज्य मंत्री, उपमंत्री। उनके बीच का अंतर है, उनका पदक्रम ,उनका वेतन तथा राजनैतिक महत्त्व। अतः कथन 1 सही नही है।
    • “प्रधानमंत्री सहित मंत्रिपरिषद के सदस्यों की कुल संख्या, लोकसभा की कुल संख्या की 15 प्रतिशत से अधिक नही होगी” इस उपबंध का समावेश 91वें संविधान संसोधन विधेयक 2003 द्वारा किया गया था। अतः कथन 2 सही है।

    85. निम्नलिखित में कौन-सी लोक सभा की अनन्य शक्ति(याँ) है/हैं?

    1. आपात की उद्घोषणा का अनुसमर्थन करना
    2. मंत्रिपरिषद के विरुद्ध अविश्वास प्रस्ताव पारित करना
    3. भारत के राष्ट्रपति पर महाभियोग चलाना

    नीचे दिए गए कूट का प्रयोग कर सही उत्तर चुनिए:

    (a) 1 और 2

    (b) केवल 2

    (c) 1 और 3

    (d) केवल 3

    उत्तर: (b) 

    व्याख्या:

    • मंत्रिपरिषद के विरुद्ध अविश्वास प्रस्ताव पारित करना लोकसभा की अनन्य शक्ति है। जब लोकसभा मंत्रिपरिषद के विरुद्ध एक अविश्वास प्रस्ताव पारित करती है तो सभी मंत्रियों (जिसमे राज्य सभा के मंत्री भी शामिल हों) को त्याग पत्र देना पड़ता है। अतः कथन 2 सही है।
    • आपात की उद्घोषणा का संकल्प संसद के प्रत्येक सदन की कुल सदस्य संख्या के बहुमत तथा उपस्थित व मतदान करने वाले सदस्यों के 2/3 बहुमत द्वारा पारित किया जाना आवश्यक होगा। राष्ट्रीय आपात की घोषणा को संसद के प्रत्येक सदन के समक्ष रखा जाता है तथा एक महीने के अंदर अनुमोदन न मिलने पर यह प्रवर्तन में नहीं रहती, किंतु एक बार अनुमोदन मिलने पर छह माह के लिये प्रवर्तन में बनी रह सकती है। परिणामतः आपात की उद्घोषणा का अनुसमर्थन लोकसभा तथा राज्यसभा दोनों द्वारा किया जाता है। यह लोकसभा की अनन्य शक्ति नहीं है। अतः कथन 1 सही नहीं है।
    • राष्ट्रपति पर महाभियोग का प्रस्ताव मूल सदन (Originating House) में विशेष बहुमत (दो-तिहाई) द्वारा पारित किया जाना चाहिये। इसके बाद प्रस्ताव को विचार हेतु दूसरे सदन में भेजा जाता है। दूसरा सदन एक निरीक्षक के रूप में कार्य करता है। राष्ट्रपति पर लगे आरोपों की जांँच के लिये एक प्रवर समिति का गठन किया जाता है। भारत के राष्ट्रपति पर महाभियोग चलाना लोकसभा तथा राज्यसभा दोनों के अंतर्गत आता है। यह लोकसभा की अनन्य शक्ति नहीं है। अतः कथन 3 सही नहीं है।

    86. भारत में दल-बदल विरोधी कानून के सन्दर्भ में, निम्नलिखित कथनों पर विचार कीजिये :

    1. यह कानून विनिर्दिष्ट करता है कि कोई नामनिर्दिष्ट विधायक सदन में नियुक्त होने के छह मास के अन्दर किसी राजनीतिक दल में शामिल नहीं हो सकता।
    2. यह कानून कोई समयावधि नहीं देता जिसके अन्दर पीठासीन अधिकारी को दल-बदल मामला विनिश्चित करना होता है।

    उपर्युक्त कथनों में कौन-सा/से सही है/हैं?

    (a) केवल 1

    (b) केवल 2

    (c) 1 और 2 दोनों 

    (d) न तो 1, न ही 2

    उत्तर: (b) 

    व्याख्या:

    • दल-बदल विरोधी कानून के तहत किसी जनप्रतिनिधि को अयोग्य घोषित किया जा सकता है यदि:
      • एक निर्वाचित सदस्य स्वेच्छा से किसी राजनीतिक दल की सदस्यता छोड़ देता है।
      • कोई निर्दलीय निर्वाचित सदस्य किसी राजनीतिक दल में शामिल हो जाता है।
      • किसी सदस्य द्वारा सदन में पार्टी के पक्ष के विपरीत वोट किया जाता है।
      • कोई सदस्य स्वयं को मतदान से अलग रखता है।
      • छह महीने की समाप्ति के बाद कोई मनोनीत सदस्य किसी राजनीतिक दल में शामिल हो जाता है।
    • अतः कोई भी नामनिर्दिष्ट विधायक सदन में नियुक्त होने के छः माह के भीतर किसी राजनितिक दल में शामिल हो सकता है परंतु छः माह के उपरांत नहीं। अतः कथन 1 गलत है।
    • कानून के अनुसार, सदन के अध्यक्ष के पास सदस्यों को अयोग्य घोषित करने संबंधी निर्णय लेने की शक्ति है।
    • यदि सदन के अध्यक्ष के दल से संबंधित कोई शिकायत प्राप्त होती है तो सदन द्वारा चुने गए किसी अन्य सदस्य को इस संबंध में निर्णय लेने का अधिकार है। अतः यह कानून कोई समयावधि नहीं निर्धारित करता जिसके भीतर पीठासीन अधिकारी को दल-बदल का मामला विनिश्चित करना होता है। अतः कथन 2 सही है।

    87. निम्नलिखित कथनों पर विचार कीजिये:

    1. भारत का महान्यायवादी और भारत का सॉलिसिटर जनरल ही सरकार के एकमात्र अधिकारी हैं जिन्हें भारत की संसद की बैठकों में भाग लेने की अनुमति है।
    2. भारत के संविधान के अनुसार, भारत का महान्यायवादी अपना त्यागपत्र दे देता है, जब वह सरकार जिसने उसको नियुक्त किया था इस्तीफा देती है।

    उपर्युक्त कथनों में कौन-सा/से सही है/हैं?

    (a) केवल 1

    (b) केवल 2

    (c) 1 और 2 दोनों 

    (d) न तो 1, न ही 2

    उत्तर: (d) 

    व्याख्या:

    • संविधान के अनुसार, भारत के महान्यायवादी को वोट देने के अधिकार के बिना उसे संसद के दोनों सदनों या उनकी संयुक्त बैठक और संसद की किसी भी समिति की कार्यवाही में बोलने तथा भाग लेने का अधिकार है, जिसका वह सदस्य नामित किया जाता है। वह उन सभी विशेषाधिकारों और उन्मुक्तियों का हकदार होता है जो एक संसद सदस्य को प्राप्त होते हैं। वह सरकारी सेवकों की श्रेणी में नहीं आता है, अत: उसे निजी कानूनी अभ्यास से वंचित नहीं किया जाता है। हालाँकि उसे भारत सरकार के खिलाफ किसी मामले में सलाह या संक्षिप्त जानकारी देने का अधिकार नहीं है।
    • भारत का महान्यायवादी (Solicitor General of India) और भारत के अतिरिक्त महान्यायवादी (Additional Solicitor General) आधिकारिक ज़िम्मेदारियों को पूरा करने में महान्यायवादी की सहायता करते हैं। अतः ऐसा वर्णित नहीं है की भारत का महान्यायवादी भी भारत की संसद की बैठकों में भाग लेता है। अतः कथन 1 सही नहीं है।
    • महान्यायवादी को हटाने की प्रक्रिया और आधार संविधान में नहीं बताए गए हैं। वह राष्ट्रपति के प्रसादपर्यंत पद धारण करता है (राष्ट्रपति द्वारा किसी भी समय हटाया जा सकता है)। अतः भारत का महान्यायवादी सरकार के इस्तीफा देने पर त्यागपत्र नहीं देता है। अतः कथन 2 सही नहीं है।

    https://www.drishtiias.com/hindi/daily-updates/daily-news-analysis/attorney-general-of-india

    88. भारत के न्यायालयों द्वारा जारी रिटों के सन्दर्भ में, निम्नलिखित कथनों पर विचार कीजिये :

    1. किसी प्राइवेट संगठन के विरुद्ध, जब तक कि उसको कोई सार्वजनिक कार्य नहीं सौंपा गया हो, परमादेश (मैंडेमस) नहीं होगा।
    2. किसी कंपनी के विरुद्ध, भले ही वह कोई सरकारी कंपनी हो, परमादेश (मैंडेमस) नहीं होगा।
    3. कोई भी लोक-प्रवण व्यक्ति (पब्लिक माइंडेड परसन) अधिकार-पृच्छा (क्वो वारंटो) रिट प्राप्त करने हेतु न्यायालय में समावेदन करने के लिए याची (पिटीशनर) हो सकता है।

    उपर्युक्त कथनों में कौन-से सही हैं?

    (a) केवल 1 और 2

    (b) केवल 2 और 3

    (c) केवल 1 और 3

    (d) 1, 2 और 3

    उत्तर: (c) 

    व्याख्या:

    • परमादेश (मैंडेमस) न्यायालय से एक आदेश के रूप में एक न्यायिक उपाय है। यह किसी प्राधिकरण को वैधानिक प्रावधान के खिलाफ कुछ करने के लिये मजबूर करने के लिये जारी नहीं किया जा सकता है। उदाहरण के लिये, इसका उपयोग निचली अदालत में किये गए आवेदनों पर एक विशिष्ट कार्रवाई करने हेतु मजबूर करने के लिये नहीं किया जा सकता है, लेकिन अगर अदालत एक या दूसरे तरीके से शासन करने से इनकार करती है तो अदालत को आदेश देने के लिये एक परमादेश का उपयोग किया जा सकता है। 
    • अतः किसी प्राइवेट संगठन के विरुद्ध जब तक कि उसको कोई सार्वजनिक कार्य नहीं सौपा गया हो, परमादेश जारी नहीं होगा। अतः विकल्प 1 सत्य है।
    • परमादेश किसी भी सरकार, अधीनस्थ न्यायालय, निगम या सार्वजनिक प्राधिकरण को करने के लिये (या इससे मना करना) कुछ विशिष्ट कार्य जो वह निकाय कानून के तहत करने के लिये बाध्य है (या करने से बचना) और जो सार्वजनिक कर्तव्य की प्रकृति में है और कुछ मामलों में एक वैधानिक कर्तव्य में से एक है। परमादेश में स्पष्ट वर्णित है कि यह किसी सार्वजनिक निगम / प्राधिकरण (सरकारी कंपनी) के विरुद्ध जारी किया जा सकता है। अतः विकल्प 2 गलत है।
    • अधिकार पृच्छा का अर्थ ‘आप क्या प्राधिकार है?’ होता है यह अवैधानिक रूप से किसी सार्वजनिक पद पर आसीन व्यक्ति के विरुद्ध जारी किया जाता है।
      • सर्वोच्च न्यायालय या उच्च न्यायालय किसी व्यक्ति द्वारा सार्वजनिक कार्यालय के अवैध हड़पने को रोकने के लिये यह रिट जारी करता है। इस रिट के माध्यम से, अदालत किसी व्यक्ति के सार्वजनिक कार्यालय के दावे की वैधता की जांँच करती है।
      • यह तभी जारी किया जा सकता है जब किसी कानून या संविधान द्वारा सृजित स्थायी स्वरूप का वास्तविक सार्वजनिक कार्यालय शामिल हो।
      • इसे निजी या मंत्रिस्तरीय कार्यालय के खिलाफ जारी नहीं किया जा सकता है।
      • यह रिट पीड़ित व्यक्ति के अलावा किसी अन्य व्यक्ति को निवारण की मांग करने का अधिकार देता 
      • है। अतः कोई भी लोक-प्रवण व्यक्ति अधिकार पृच्छा रिट प्राप्त करने हेतु न्यायालय में समावेदन करने के लिये याची हो सकता है। अतः विकल्प 3 सत्य है।

    https://www.drishtiias.com/hindi/paper2/right-to-constitutional-remedy-and-importance

    89. आयुष्मान भारत डिजिटल मिशन के सन्दर्भ में, निम्नलिखित कथनों पर विचार कीजिये:

    1. प्राइवेट अस्पतालों और सरकारी अस्पतालों को इसे अवश्य अपनाना चाहिए।
    2. चूँकि इसका लक्ष्य स्वास्थ्य की सर्वजनीन व्याप्ति है, अंततोगत्वा भारत के हर नागरिक को इसका हिस्सा हो जाना चाहिए।
    3. यह पूरे देश में निर्बाध रूप से लागू किया जा सकता है।

    उपर्युक्त कथनों में कौन-सा/से सही है/हैं?

    (a) केवल 1 और 2

    (b) केवल 3

    (c) केवल 1 और 3

    (d) 1, 2 और 3

    उत्तर: (d) 

    व्याख्या:

    इस मिशन के तहत नागरिक अपना आयुष्मान भारत स्वास्थ्य खाता संख्या प्राप्त कर सकेंगे, जिससे उनके डिजिटल स्वास्थ्य रिकॉर्ड को जोड़ा जा सकेगा।

    • आयुष्मान भारत देश की एक प्रमुख योजना है, जिसे सार्वभौमिक स्वास्थ्य कवरेज (UHC) के दृष्टिकोण को प्राप्त करने हेतु राष्ट्रीय स्वास्थ्य नीति 2017 की सिफारिश के अनुसार शुरू किया गया था।
    • इसका उद्देश्य सभी भारतीय नागरिकों को अस्पतालों, बीमा फर्मों और नागरिकों को आवश्यकता पड़ने पर इलेक्ट्रॉनिक रूप से स्वास्थ्य रिकॉर्ड तक पहुँचने में मदद करने हेतु डिजिटल स्वास्थ्य आईडी प्रदान करना है।
    • मिशन के पायलट प्रोजेक्ट की घोषणा प्रधानमंत्री ने 15 अगस्त, 2020 को लाल किले की प्राचीर से की थी।
    • यह पायलट परियोजना छह राज्यों और केंद्रशासित प्रदेशों में चरणबद्ध रूप में लागू की जा रही है
    • इसकी कार्यान्वयन एजेंसी स्वास्थ्य और परिवार कल्याण मंत्रालय के तहत स्थापित राष्ट्रीय स्वास्थ्य प्राधिकरण (NHA) होगी।
    • यह प्रत्येक नागरिक को प्रदान किया जाएगा जो उनके स्वास्थ्य खाते के रूप में भी काम करेगा। इस स्वास्थ्य खाते में प्रत्येक परीक्षण, प्रत्येक बीमारी, डॉक्टर से अपॉइंटमेंट, ली गई दवाओं और निदान का विवरण होगा। 
    • स्वास्थ्य आईडी निःशुल्क व स्वैच्छिक है। यह स्वास्थ्य डेटा का विश्लेषण करने में मदद करेगी और स्वास्थ्य कार्यक्रमों के बेहतर नियोजन, बजट तथा कार्यान्वयन सुनिश्चित करेगा।

    अतः कथन 1, 2 और 3 सही है।

    https://www.drishtiias.com/hindi/daily-updates/daily-news-analysis/ayushman-bharat-digital-mission-1%5D

    90. लोकसभा के उपाध्यक्ष के सन्दर्भ में, निम्नलिखित कथनों पर विचार कीजिये :

    1. लोकसभा के कार्य-पद्धति और कार्य-संचालन नियमों के अनुसार, उपाध्यक्ष का निर्वाचन उस तारीख को होगा जो अध्यक्ष नियत करे।
    2. यह आज्ञापक उपबंध है कि लोकसभा के उपाध्यक्ष के रूप में किसी प्रतियोगी का निर्वाचन या तो मुख्य विपक्षी दल से, या शासक दल से, होगा।
    3. सदन की बैठक की अध्यक्षता करते समय उपाध्यक्ष की शक्ति वैसी ही होती है जैसी कि अध्यक्ष की, और उसके विनिर्णयों के विरुद्ध अपील नहीं हो सकती।
    4. उपाध्यक्ष की नियुक्ति के बारे में सुस्थापित संसदीय पद्धति यह है कि प्रस्ताव अध्यक्ष द्वारा रखा जाता है और प्रधान मंत्री द्वारा विधिवत समर्थित होता है।

    उपर्युक्त कथनों में कौन-से सही हैं?

    (a) केवल 1 और 3

    (b) 1, 2 और 3

    (c) केवल 3 और 4

    (d) केवल 2 और 4

    उत्तर: (a) 

    व्याख्या:

    • लोकसभा में प्रक्रिया और कार्य संचालन के नियमों के अनुसार, "उपसभापति का चुनाव उस तारीख को होगा जो अध्यक्ष तय कर सकता है। कन्वेंशन के अनुसार, डिप्टी स्पीकर का पद भारत में विपक्षी दल को दिया जाता है। अतः विकल्प 1 सही है।
    • 10वीं लोकसभा तक अध्यक्ष और उपाध्यक्ष दोनों आमतौर पर सत्ताधारी दल से चुने जाते थे। 11वीं लोकसभा के बाद से यह आम सहमति रही है कि अध्यक्ष का पद सत्ता पक्ष/गठबंधन को और उपाध्यक्ष का पद मुख्य विपक्षी दल को दिया जाता है। अतः विकल्प 2 सही है।
    • वह सदन के अंदर भारत के संविधान के प्रावधानों, लोकसभा की प्रक्रिया और कार्य संचालन के नियमों तथा संसदीय मामलों का अंतिम व्याख्याकार होता है। वह इन प्रावधानों की व्याख्या के मामलों में अक्सर ऐसे निर्णय देता है जिनका सदस्यों द्वारा सम्मान किया जाता है और जो प्रकृति में बाध्यकारी होते हैं। अर्थात् उसका निर्णय अंतिम होता है। अनुच्छेद 95 उपाध्यक्ष या अन्य व्यक्ति को अध्यक्ष के पद के कर्तव्यों का पालन करने या अध्यक्ष के रूप में कार्य करने की शक्ति का प्रावधान प्रदान करता है। अतः उपाध्यक्ष द्वारा सदन की बैठक की अध्यक्षता करते समय उसकी शक्ति ठीक लोकसभा अध्यक्ष के समान होती है और उसके विनिर्णयों के विरुद्ध कोई अपील नहीं हो सकती है। अतः विकल्प 3 सत्य है।
    • उपाध्यक्ष का चुनाव लोकसभा द्वारा ही अपने सदस्यों में से किया जाता है। स्पीकर के चुनाव के बाद डिप्टी स्पीकर का भी चुनाव होता है। अतः विकल्प 4 असत्य है

    https://www.drishtiias.com/hindi/paper2/the-office-of-speaker-of-lok-sabha 

    https://www.drishtiias.com/hindi/daily-updates/daily-news-analysis/election-of-speaker-and-deputy-speaker

    91. ‘‘त्वरित वित्तीयन प्रपत्र (Rapid Financing Instrument)’’ और ‘‘त्वरित ऋण सुविधा (Rapid Credit Facility)’’, निम्नलिखित में किस एक के द्वारा उधार दिए जाने के उपबंधों से संबंधित हैं?

    (a) एशियाई विकास बैंक

    (b) अंतर्राष्ट्रीय मुद्रा कोष

    (c) संयुक्त राष्ट्र पर्यावरण कार्यक्रम वित्त पहल

    (d) विश्व बैंक

    उत्तर: B

    व्याख्या:

    रैपिड फाइनेंसिंग इंस्ट्रूमेंट (RFI)

    • रैपिड फाइनेंसिंग इंस्ट्रूमेंट (RFI) त्वरित वित्तीय सहायता प्रदान करता है, जो भुगतान संतुलन की तत्काल आवश्यकता का सामना करने वाले सभी सदस्य देशों के लिये उपलब्ध है। सदस्य देशों की विविध आवश्यकताओं को पूरा करने के लिये IMF की वित्तीय सहायता को और अधिक लचीला बनाने हेतु RFI को व्यापक सुधार के हिस्से के रूप में बनाया गया था। RFI ने IMF की पिछली आपातकालीन सहायता नीति को प्रतिस्थापित किया है और इसका उपयोग कई तरह की परिस्थितियों में किया जा सकता है।

    रैपिड क्रेडिट सुविधा (RCF)

    • रैपिड क्रेडिट सुविधा (RCF) निम्न आय वाले देशों (LIC) को तत्काल भुगतान संतुलन (BoP) की आवश्यकता प्रदान करती है, जिसमें कोई पूर्व-पोस्ट शर्त नहीं है, जहाँ एक पूर्ण आर्थिक कार्यक्रम तो आवश्यक है और ही व्यवहार्य। RCF की स्थापना एक व्यापक सुधार के हिस्से के रूप में की गई थी ताकि फंड की वित्तीय सहायता को अधिक लचीला बनाया जा सके तथा संकट के समय सहित LIC की विविध आवश्यकताओं के अनुरूप बेहतर बनाया जा सके।
    • RCF के तहत तीन क्षेत्र हैं
      • घरेलू अस्थिरता, आपात स्थिति और नाजुकता जैसे स्रोतों की एक विस्तृत शृंखला के कारण तत्काल BoP ज़रूरतों के लिये एक "नियमित खिड़की
      • अचानक, बहिर्जात झटके की वज़ह से तत्काल BoP जरूरतों के लिये एक "बहिर्जात शॉक विंडो" और
      • प्राकृतिक आपदाओं के कारण होने वाली तत्काल BoP ज़रूरतों के लिये एक "बड़ी प्राकृतिक आपदा खिड़की" जहाँ क्षति सदस्य के सकल घरेलू उत्पाद के 20% के बराबर या उससे अधिक होने का अनुमान है

    स्रोत: IMF

    92. भारतीय अर्थव्यवस्था के सन्दर्भ में, निम्नलिखित कथनों पर विचार कीजिये :

    1. अंकित प्रभावी विनिमय दर (Nominal Effective Exchange Rate – NEER) में वृद्धि रुपए की मूल्यवृद्धि को दर्शाता है।
    2. वास्तविक प्रभावी विनिमय दर (Real Effective Exchange Rate – REER) में वृद्धि व्यापार प्रतिस्पर्धात्मकता में सुधार को दर्शाता है।
    3. अन्य देशों में मुद्रास्फीति के सापेक्ष घरेलू मुद्रास्फीति में बढ़ने की प्रवृत्ति NEER और REER के बीच में वर्धमान अपसरण उत्पन्न कर सकता है।

    उपर्युक्त कथनों में कौन-से सही हैं?

    (a) केवल 1 और 2

    (b) केवल 2 और 3

    (c) केवल 1 और 3

    (d) 1, 2 और 3

    उत्तर: C

    • भारतीय रिज़र्व बैंक (Reserve Bank of India-RBI) 36 व्यापारिक साझेदार देशों की मुद्राओं के संबंध में रुपए की नॉमिनल इफेक्टिव एक्सचेंज रेट’ (Nominal Effective Exchange Rate-NEER) को सारणीबद्ध करता है।
      • यह एक प्रकार का भारित सूचकांक है अर्थात् इसमें उन देशों को अधिक महत्त्व दिया जाता है, जिनके साथ भारत अधिक व्यापार करता है।
      • इस सूचकांक में कमी रुपए के मूल्य में ह्रास को दर्शाती है, जबकि सूचकांक में बढ़ोतरी रुपए के मूल्य में अभिमूल्यन को दर्शाती है।
    • RBI द्वारा जारी NEER के अनुसार, बीते कुछ समय में रुपया नवंबर 2018 के पश्चात् से अपने सबसे निचले स्तर पर पहुँच गया है।
    • NEER के अतिरिक्त रियल इफेक्टिव एक्सचेंज रेट’ (Real Effective Exchange Rate-REER) भी भारतीय अर्थव्यवस्था में हो रहे परिवर्तनों को मापने के लिये एक महत्त्वपूर्ण मापदंड है।
      • REER के अंतर्गत NEER में शामिल अन्य कारकों के अतिरिक्त विभिन्न अर्थव्यवस्थाओं में घरेलू मुद्रास्फीति को भी ध्यान में रखा जाता है, जिसके कारण इसका महत्त्व अधिक बढ़ जाता है।
    • NEER विदेशी मुद्राओं के संदर्भ में घरेलू मुद्रा के द्विपक्षीय विनिमय दरों का भारित औसत होता है। जबकि REER मुद्रास्फीति के प्रभावों के लिये समायोजित अन्य प्रमुख मुद्राओं के सापेक्ष घरेलू मुद्रा का भारित औसत है।
    • NEER विदेशी मुद्रा बाज़ार के संदर्भ में देश की अंतर्राष्ट्रीय प्रतिस्पर्द्धा का एक संकेतक है।
    • REER की गणना NEER में मूल्य परिवर्तन को समायोजित करने के पश्चात् की जाती है। इस प्रकार अर्थशास्त्री NEER की अपेक्षा REER को अधिक महत्त्व देते हैं।
    • NEER = विशेष आहरण अधिकार (SDR) के संदर्भ में घरेलू विनिमय दर/विशेष आहरण अधिकार (SDR) के संदर्भ में विदेशी विनिमय दर
    • REER = NEER  × (घरेलू मूल्य सूचकांक/विदेशी मूल्य सूचकांक). अतः कथन 1,  और 3 सही हैं

    https://www.drishtiias.com/hindi/daily-updates/daily-news-analysis/exchange-rate-of-rs

    93. भारतीय अर्थव्यवस्था के सन्दर्भ में, निम्नलिखित कथनों पर विचार कीजिये :

    1. यदि मुद्रास्फीति अत्यधिक है, तो भारतीय रिज़र्व बैंक (RBI) सम्भावित रूप से सरकारी प्रतिभूतियाँ खरीद सकता है।
    2. यदि रुपए का तेज़ी से मूल्यह्रास हो रहा है, तो RBI बाज़ार में डॉलरों का सम्भावित रूप से विक्रय कर सकता है।
    3. यदि USA या यूरोपीय संघ में ब्याज दरें गिरती होतीं, तो इससे सम्भावित रूप से RBI की डॉलरों की खरीद प्रेरित हो सकती है।

    उपर्युक्त कथनों में कौन-से सही हैं?

    (a) केवल 1 और 2

    (b) केवल 2 और 3

    (c) केवल 1 और 3

    (d) 1, 2 और 3

    उत्तर: B

    व्याख्या:

    • मुद्रास्फीति को नियंत्रित करने के लिये, भारतीय रिज़र्व बैंक मुद्रा बाज़ार में प्रतिभूतियों को बेचता है जो बाज़ार से अतिरिक्त तरलता को अवशोषित करता है।
    • जब रुपए का मूल्य तेज़ी से गिरता है तो रिज़र्व बैंक डॉलर की आपूर्ति को बढ़ा देता है अमेरिका या यूरोपीय संघ जैसे अंतर्राष्ट्रीय बाज़ारों में जैसी ही ब्याज दरें गिरती है तो उसका प्रभाव भारत जैसे अन्य देशों के बाज़ारों में डॉलर की खरीद पर पड़ता है।

    94. ‘‘G20 कॉमन प्रेमवर्क’’ के सन्दर्भ में, निम्नलिखित कथनों पर विचार कीजिये:

    1. यह G20 और उसके साथ पेरिस क्लब द्वारा समर्थित पहल है।
    2. यह अधारणीय ऋण वाले निम्न आय देशों को सहायता देने की पहल है।

    उपर्युक्त कथनों में कौन-सा/से सही है/हैं?

    (a) केवल 1

    (b) केवल 2

    (c) 1 और 2 दोनों

    (d) न तो 1, न ही 2

    उत्तर:C

    व्याख्या:

    • DSSI (कॉमन फ्रेमवर्क) से परे ऋण उपचार के लिये सामान्य ढाँचा, पेरिस क्लब के साथ मिलकर G20 द्वारा समर्थित एक पहल है। जिसका उद्देश्य संरचनात्मक तरीके से, कम आय वाले देशों को अस्थिर (Unsustainable) ऋण के साथ समर्थनकरना है।

    95. भारतीय अर्थव्यवस्था के सन्दर्भ में, ‘‘मुद्रास्फीति-सहलग्न बॉन्ड (Inflation-Indexed Bonds – IIBs)’’ के क्या लाभ हैं?

    1. सरकार IIBs के रूप में अपने ऋणग्रहण पर कूपन दरों को कम कर सकती है।
    2. IIBs निवेशकों को मुद्रास्फीति के बारे में अनिश्चितता से सुरक्षा प्रदान करते हैं।
    3. IIBs पर प्राप्त ब्याज और साथ ही साथ पूँजीगत लाभ कर-योग्य नहीं होते।

    उपर्युक्त कथनों में कौन-से सही हैं?

    (a) केवल 1 और 2  

    (b) केवल 2 और 3

    (c) केवल 1 और 3

    (d) 1, 2 और 3

    उत्तर: (a)

    व्याख्या:

    रिज़र्व बैंक ऑफ़ इंडिया के अनुसार,

    • 1997 में जारी CIB ने केवल मूलधन को मुद्रास्फीति संरक्षण प्रदान किया, न कि ब्याज भुगतान को। जबकि IIB के नए उत्पाद मूलधन और ब्याज भुगतान दोनों को मुद्रास्फीति सुरक्षा प्रदान करेंगे।
    • मुद्रास्फीति के खिलाफ समायोजित मूलधन पर निश्चित कूपन दर का भुगतान करके ब्याज दर को मुद्रास्फीति से सुरक्षा प्रदान की जाएगी तथा सरकार IIB के रूप में अपने ऋण ग्रहण पर कूपन दरों को कम कर सकती है। अतः कथन 1 और 2 सही हैं। 
    • मुद्रास्फीति अनुक्रमित बॉण्ड (आईआईबी) 1997 के दौरान पूंजी अनुक्रमित बॉण्ड (सीआईबी) के नाम पर जारी किये गए थे।तथा आईआईबी का द्वितीयक बाज़ार (बीएसई, एनएसई, और अन्य स्टॉक एक्सचेंजों के माध्यम से) में भी कारोबार किया जा सकता है, हालांकि, यदि वे द्वितीयक बाज़ार में बेचे जाते हैं और लाभ कमाया जाता है, तो पूंजीगत लाभ कर का भुगतान करना होता  है। अत: कथन 3 सही नहीं है।

    96. भारत में कार्य कर रही विदेशी-स्वामित्व की e-वाणिज्य फर्मों के सन्दर्भ में, निम्नलिखित कथनों में कौन-सा/से सही है/हैं?

    1. अपने प्लेटफॉर्मों को बाज़ार-स्थान के रूप में प्रस्तुत करने के अतिरिक्त वे स्वयं अपने माल का विक्रय भी कर सकते हैं।
    2. वे अपने प्लेटफॉर्मों पर किस अंश तक बड़े विक्रेताओं को स्वीकार कर सकते हैं, यह सीमित है।

    नीचे दिए कूट का प्रयोग कर सही उत्तर चुनिए:

    (a) केवल 1

    (b) केवल 2

    (c) 1 और 2 दोनों 

    (d) न तो 1, न ही 2

    उत्तर: (b) 

    व्याख्या:

    e-वाणिज्य की नियमावली के आधार पर,

    • बाज़ार-स्थान प्रदान करने वाली ई-कॉमर्स संस्थाएँ इन्वेंट्री यानी बेची जाने वाली वस्तुओं पर स्वामित्व का प्रयोग नहीं करेंगी। इन्वेंट्री पर इस तरह का स्वामित्व व्यवसाय को एक इन्वेंट्री आधारित मॉडल में प्रस्तुत करेगा। अतः कथन 1 सही नहीं है।
    • एक ई-वाणिज्य इकाई अपने बाज़ार-स्थान के माध्यम से प्रभावित बिक्री के 25% से अधिक एक विक्रेता को अनुमति नहीं देगी।अतः इसकी सीमा का निर्धारण किया गया है। अतः कथन 2 सही है।

    97. निम्नलिखित में कौन-कौन-से कार्यकलाप अर्थव्यवस्था में वास्तविक क्षेत्रक (रियल सेक्टर) का निर्माण करते हैं?

    1. किसानों को अपनी फसलें काटना
    2. कपड़ा मिलों का कच्चे कपास को कपड़े में बदलना
    3. किसी वाणिज्यिक बैंक का किसी व्यापारी कंपनी को धनराशि उधार देना
    4. किसी कॉर्पोरेट निकाय का विदेश में रुपया-अंकित मूल्य के बॉन्ड जारी करना

    नीचे दिए कूट का प्रयोग कर सही उत्तर चुनिए:          

    (a) केवल 1 और 2

    (b)  केवल 2, 3 और 4

    (c) केवल 1, 3 और 4

    (d)  1, 2, 3 और 4

    उत्तर: (a)

    व्याख्या:

    • अर्थव्यवस्था के वास्तविक क्षेत्रकों  में उद्यम (गैर-वित्तीय निगम), घरेलू और गैर-लाभकारी संस्थाएँ शामिल हैं, जो परिवारों की सेवा करती हैं। अतः केवल कथन 1 और 2 सही हैं।

    98. भारत के सन्दर्भ में हाल ही में जनसंचार-माध्यमों में अक्सर चर्चित ‘‘अप्रत्यक्ष अंतरण’’ को निम्नलिखित में कौन-सी एक स्थिति सर्वोत्तम रूप से प्रतिबिंबित करती है?

    (a) कोई भारतीय कंपनी, जिसने किसी विदेशी उद्यम में निवेश किया हो और अपने निवेश पर मिलने वाले लाभ पर उस बाहरी देश को कर अदा करती हो

    (b) कोई विदेशी कंपनी, जिसने भारत में निवेश किया हो और अपने निवेश से मिलने वाले लाभ पर अपने आधारभूत देश को कर अदा करती हो

    (c) कोई भारतीय कंपनी, जो किसी बाहरी देश में मूर्त संपत्ति खरीदती है और उनका मूल्य बढ़ने पर उन्हें बेच देती है तथा प्राप्ति को भारत में अंतरित कर देती है

    (d) कोई विदेशी कंपनी शेयर अंतरित करती है और ऐसे शेयर भारत में स्थित परिसंपत्तियों से अपना वस्तुगत मूल्य व्युत्पन्न करते हैं

    उत्तर: (d)

    व्याख्या:

    • जन संचार माध्यमों में अक्सर चर्चित शब्द ‘अप्रत्यक्ष हस्तांतरण’ उन स्थितियों को संदर्भित करता है जहाँ विदेशी संस्थाएँ भारत में शेयर या संपत्ति रखती हैं, ऐसी विदेशी संस्थाओं के शेयरों को भारत में अंतर्निहित परिसंपत्तियों के प्रत्यक्ष हस्तांतरण के बजाय स्थानांतरित किया जाता है।

    अतः विकल्प (d) सर्वाधिक उपयुक्त है।

    99. किसी संगठन या कंपनी द्वारा किए गए व्यय के सन्दर्भ में, निम्नलिखित कथनों में कौन-सा/से सही है/हैं?

    1. नई प्रौद्योगिकी प्राप्त करना पूँजीगत व्यय है।
    2. ऋण वित्तीयन को पूँजीगत व्यय माना जाता है, जबकि ईक्विटी वित्तीयन को राजस्व व्यय माना जाता है।

    नीचे दिए कूट का प्रयोग कर सही उत्तर चुनिए:

    (a) केवल 1

    (b) केवल 2

    (c) 1 और 2 दोनों

    (d) न तो 1, न ही 2

    उत्तर: (a)

    व्याख्या:

    • पूंजीगत व्यय (CapEx) एक कंपनी द्वारा संपत्ति, पौधों, भवनों, प्रौद्योगिकी, या उपकरण जैसी भौतिक संपत्तियों को प्राप्त करने, अपग्रेड करने और बनाए रखने के लिये उपयोग की जाने वाली धनराशि है। CapEx का उपयोग अक्सर किसी कंपनी द्वारा नई परियोजनाएँ या निवेश करने के लिये किया जाता है। अतः कथन 1 सही है।
    • कथन 2 सही नहीं है, क्योंकि इक्विटी वित्तीयन किसी कंपनी के स्टॉक को नकदी के बदले में बेचकर किसी संगठन की तरलता की ज़रूरतों को पूरा करने के लिये धन जुटाने की एक विधि है, इसलिये इसे पूंजीगत व्यय में माना जाता है।

    100. भारतीय अर्थव्यवस्था के सन्दर्भ में, निम्नलिखित कथनों पर विचार कीजिये :

    1. घरेलू वित्तीय बचत का एक भाग सरकारी ऋणग्रहण के लिए जाता है।
    2. नीलामी में बाज़ार-संबंधित दरों पर जारी दिनांकित प्रतिभूतियाँ, आंतरिक ऋण का एक बड़ा घटक होती हैं।

    उपर्युक्त कथनों में कौन-सा/से सही है/हैं?

    (a) केवल 1

    (b) केवल 2

    (c) 1 और 2 दोनों 

    (d) न तो 1, न ही 2

    उत्तर: (c)

    व्याख्या:

    सरकारी ऋण स्स्टेटस पत्र द्वारा, निम्नलिखित खंड आंतरिक ऋण के घटकों का विवरण प्रदान करता है।

    • बाज़ार ऋण - दिनांकित प्रतिभूतियां  राजकोषीय घाटे के वित्तपोषण के लिये उपयोग किये जाने वाले प्रमुख साधन हैं। वे प्रत्येक वित्तीय वर्ष में क्रमशः अप्रैल-सितंबर और अक्तूबर-मार्च को कवर करने वाले दो छमाही जारी कैलेंडर के अनुसार नीलामी के माध्यम से जारी किये जाते हैं। अतःकथन 2 सही है।
    close
    एसएमएस अलर्ट
    Share Page
    images-2
    images-2
    × Snow